Retirement Planning and Employee Benefits

¡Supera tus tareas y exámenes ahora con Quizwiz!

Pete died recently and held a profit-sharing account balance of $300,000. As part of Pete's account allocation, he maintained a universal life insurance policy with a death benefit of $100,000. The cash value of the policy is $20,000 and Pete had paid total P.S. 58 costs of $3,000. What is the current tax treatment of a lump-sum distribution of the profit-sharing account and life insurance proceeds?

$317,000 ordinary income and $83,000 tax-free. The $300,000 account balance plus the life insurance cash value, less the P.S. 58 costs paid of $17,000 is ordinary income. The balance of the life insurance death benefit, $83,000, is tax-free.

Raul has a retirement income replacement goal of 60% of his current after-tax salary. If his current salary is $8,000 per month and 15% is assumed for taxes, what will Raul's annual income be at the beginning of his retirement?

$48,6000 is the annual after-tax income. $8,000 x 12 = $96,000 60% x $96,000 = $57,600 $57,600 x 0.85 (1 - 15% taxes) = $48,960

If the current inflation rate is 2.2% and the current rate of return is 9%, what is the inflation-adjusted rate of return?

(9 - 2.20) ÷ (1 + 0.022) = 6.65

What percentage of compensation does an employee pay up to the wage base limit to fund Social Security Benefits (i.e., retirement, disability, survivors)?

6.2% of an employee's compensation up to the wage base limit funds Social Security retirement, disability, and survivor benefits. 1.45% on unlimited compensation funds Medicare.

Evershine Inc. is setting up retirement plans for its employees. Can Evershine Inc. set up a Keogh plan?

A Keogh plan can only be set up for an unincorporated business.

Paul is a financial planner who wants to find the uses of a Keogh plan. A Keogh plan is used for which of the following reasons? (Select all that apply)

A Keogh plan is used for long-term capital accumulation, loan purposes, retirement purposes, and to shelter current earnings from federal Income tax for a self-employed individual. It is also applicable in the case of an employee of a self-employed individual.

An HR10 Keogh plan covers which of the following? (Select all that apply)

A Keogh plan only covers one or more self-employed individuals and the employees of an unincorporated business.

A Roth IRA can be rolled over to another Roth IRA:

A Roth IRA can be rolled over to another Roth IRA tax-free. There are no tax liabilities.

Which of the following statements is true in regards to a SEP tax-deferred employee retirement plan? (Select all that apply)

A SEP tax-deferred employee retirement plan is very simple to implement and not at all expensive to administer, hence making it a popular choice for employers, and even more so for employers who wish to install a tax-deferred plan and are too late to adopt a qualified plan for the year in question.

Rodney is analyzing various retirement plans. From the situations listed below, choose when a SIMPLE IRA is useful? (Select all that apply)

A SIMPLE IRA is attractive for employers who are looking for inexpensive and easy to install plans. It is ideal for employers with 100 or less employees so that the employer can fund the plan with an employee salary reduction. SIMPLE IRAs are useful for individuals with a small amount of self-employment income, and not for individuals with high self-employment income.

Which of the following statements is NOT correct regarding governmental Section 457(b) plans?

A Section 457(b) rollover distribution may be rolled over to another governmental Section 457(b) plan, a tax-advantaged employer-sponsored plan, a qualified plan, or an IRA.

How many Social Security credits must a worker earn to be eligible for retirement benefits?

A covered worker needs 40 credits to qualify for Social Security retirement benefits.

How many life insurance policies are needed to fund a cross-purchase buy-sell agreement in a partnership with 5 equal partners?

A cross-purchase buy-sell agreement in a partnership with 5 equal partners requires 20 life insurance policies. Each of the 5 partners purchases a policy insuring each of the other 4 partners (5 x 4 =20 policies).

Which of the following statements are true regarding defined benefit plans? Click all that apply.

A defined benefit plan can be designed to allow full benefits to accrue after a specified period with no further benefits accruing thereafter. In addition, a defined benefit plan makes it relatively easy to design a window plan to encourage early retirement.

Each of the following statements is correct regarding a SIMPLE IRA EXCEPT:

A distribution from a SIMPLE IRA within the first two years of participation may be subject to a 25% penalty.

Which of the following would be considered part of the Prudent Person Standard of Care? (Select all that apply)

A fiduciary must act with care, skill, prudence and diligence of a prudent person with the participant or beneficiary's best interests in mind. And they have an obligation to diversify the plan's assets to reduce the risk of loss.

Jack is looking for an annuity that will guarantee a rate of return and provide payments for his lifetime. What type of annuity and payout option should he choose?

A fixed annuity guarantees a rate of return because it puts the funds in the general account. With a single payout option, the annuitant will receive payments as long as he or she lives whether that is 1 year or 50 years.

Which of the following statements is CORRECT regarding cafeteria plans as an employee benefit?

A flexible spending account (FSA) is a type of cafeteria plan that provides very specific tax benefits and is often used even by smaller employers, including closely-held businesses. FSAs feature benefit funding through salary reductions by employees.

Which of the following statements is true regarding fully insured pension plans? Click all that apply.

A fully insured pension plan is one that is funded exclusively by life insurance or annuity contracts. There is no trusteed side fund. Such plans were once common, but the high interest rates of the late 1970s lured many pension investors away from traditional insured pension products.

A compensation arrangement that provides special severance benefits to executives if the corporation changes ownership and the covered executives are terminated is referred to as

A golden parachute plan is a compensation arrangement that provides special severance benefits to executives in the event that the corporation changes ownership and the covered executives are terminated.

Highly Compensated Top-Paid Group Key Employee

A greater than 5% percent owner or received compensation in excess of $130,000 in the prior year The group of employees in the top 20%, ranked on the basis of compensation paid for the year. A greater than 5% owner; an officer with income in excess of $185,000 or a greater than one percent owner with income greater than $150,000

Which of the following correctly describes the payment structure of a life with a 10-year certain annuity?

A life with period-certain annuity provides payments for life, but if the annuitant dies before a specified period, usually 10 to 20 years, the beneficiary will continue to receive payments until the period is over.

A money purchase Keogh plan does not have to meet minimum funding requirements. They just have to make substantial and recurring contributions, or the plan may be deemed to have terminated. State True or False.

A money purchase plan is subject to the Code's minimum funding requirements. These require the employer to make contributions to each employee's and self-employed person's account each year equal to the percentage of compensation stated in the plan.

LongLife Medical Foundation is a nongovernmental organization that wants to purchase life insurance contracts to finance Section 457 plans for its employees such that employees of LongLife do not incur any current costs. What are the conditions to be satisfied by LongLife? (Select all that apply)

A nongovernmental employer can finance a Section 457 plan with insurance contracts at no current cost to employees only if the employer retains all incidents of ownership, is the sole beneficiary and is under no obligation to transfer the policies. It is also under no obligation to pass through the proceeds of the policies.

Which of the following are reasons why a participant would NOT want to take a lump sum distribution? Click all that apply.

A participant may not want to take a limp sum distribution because he or she may be in a high tax bracket. In addition, he or she may be subject to the 20% mandatory withholding and early distribution penalties.

A person who has only investment income can contribute to an IRA.

A person needs to have earned income to be able to contribute to an IRA.

Which of the following correctly describes a qualified plan that would be deemed top-heavy?

A plan in which more than 60% of the plan benefits or contributions are for key employees is deemed top-heavy.

Which of the following statements is NOT correct regarding a profit-sharing plan?

A profit-sharing plan is a defined contribution plan under which the employer determines the amount of the contribution each year, rather than having a stated contribution obligation.

A short-term disability plan, often an insured plan, goes into effect when sick pay benefits cease, and will typically extend until?

A short-term disability plan fills the gap between sick pay and the employee's long-term disability plans. It goes into effect when the employee's sick pay benefits run out and extends until the six-month limit has been reached, when the employer's long-term disability plan, if any, and Social Security disability go into effect.

Which of the following employer-sponsored retirement plan is MOST likely to incentivize employee productivity?

A stock bonus plan is structured like a profit-sharing plan. Plan contributions are made in employer stock. Generally, seeking to increase the value of employer stock incentivizes employee productivity because an employee's individual account balance is based on the value of the employer stock.

Who bears the investment risk in a target benefit pension plan?

A target benefit pension plan is a defined contribution plan. All defined contribution plans have employee-directed individual accounts, and the employee bears the investment risk.

A qualified plan can offer a wide range of distribution options. Participants benefit from having the widest possible range of options, because this increases their flexibility in personal retirement planning. Which of the following is a disadvantage of having such a wide range of options?

A wide range of options increases administrative costs. Also, the IRS makes it difficult to withdraw a benefit option once it has been established, though this anti-cutback rule has been eased somewhat for plan years beginning after December 31, 2001. Consequently, most employers provide only a relatively limited menu of benefit forms for participants to choose from.

Amy is employed at Abacale Corporation and is a key employee. Abacale has a non-qualified plan for Amy and set aside money into it for her in 2018, 2019 and 2020. In 2021, Amy is allowed to take a withdrawal and does so. In what year may Abacale take a deduction?

Abacale Corporation may take a deduction in the year that Amy takes a distribution, or when the funds are made available, which is 2019.

Who are the fiduciary's main concerns? Click all that apply.

According to Title 29 section 1104 of the U.S Code of Law, a fiduciary shall discharge his duties with respect to a plan solely in the interest of the participants and beneficiaries.

Using the table above, after Jamel has 4 years of service his vesting will be ____ of employer matching contributions.

After Jamel has attained 4 years of service his vesting will be 60% of employer matching contributions.

What is the earliest age that a person can collect his or her own retirement benefits?

Age 62 is the earliest a person can collect Social Security retirement benefits. Those who are already receiving widow's or widower's benefits, including divorced widows or widowers, can switch to their own retirement benefits as early as age 62.

Section 119 De minimis fringe Highly compensated VEBA

All meals provided to employees on business premises are excluded from income if more than half of the employees are provided with meals. Section 132 provision where property or services are not taxable to an employee if their value is so small it makes accounting for them unreasonable. Receives compensation from the employer in excess of $130,000 (2021). A prefunded welfare benefit plan.

Which of the following plans is NOT required to provide the qualified preretirement survivor annuity (QPSA) and the qualified joint and survivor annuity (QJSA) provision?

All pension plans are required to provide the qualified preretirement survivor annuity and the qualified joint and survivor annuity provision. A Section 401(k) plan is not a pension.

Which of the following qualified plans does not require mandatory annual minimum funding?

All pension plans require annual minimum funding. A profit-sharing plan is not required to have annual minimum funding.

All qualified pension plans must provide two forms of survivorship benefits for spouses, the preretirement survivor annuity and the joint and survivor annuity. The plans that need not provide for such survivorship benefits for spouses, if the participant's nonforfeitable account balance is payable as a death benefit to that spouse, are which of the following? (Select all that apply)

All qualified pension plans, including defined benefit plans and defined contribution plans, must provide two forms of survivorship benefits for spouses, the qualified preretirement survivor annuity and the qualified joint and survivor annuity. Stock bonus plans, profit sharing plans and ESOPs generally need not provide these survivorship benefits for the spouse if the participant's nonforfeitable account balance is payable as a death benefit to that spouse.

Which of the following are alternatives to a cafeteria plan? Click all that apply.

Alternatives to cafeteria plan are flexible spending account (FSA), a fixed benefit program, and cash compensation.

Michael is self-employed and earns $40,000 in a normal year. How much would Michael normally pay in Social Security taxes?

Although Michael must pay the full FICA of 15.3%, he will receive a deduction for the employer portion. This is calculated by multiplying his income $40,000 by .9235 which results in $36,940 which is then multiplied by 15.3% to arrive at the FICA amount of $5,652 (rounded up to the nearest dollar).

Each of the following is a type of pension plan EXCEPT:

An ESOP is a type of profit-sharing plan. The other plans listed are types of pension plans.

Qualified plan fiduciaries typically include each of the following EXCEPT:

An attorney who represents the employer is not typically considered a fiduciary of a qualified plan sponsored by the employer.

An election to waive the joint and survivor form must be made before the annuity starting date. The annuity starting dates commences after what time frame?

An election to waive the joint and survivor form must be made during the 90-day period ending on the annuity starting date, that is, the date on which benefit payments should have begun to the participant.

Each of the following is considered a key employee EXCEPT:

An employee with compensation greater than $135,000 (2022) and ranked in the top 20% based on compensation is a highly compensated employee definition, not a key employee.

Which of the following statements are true in regards to employer contributions for SEP? Click all that apply.

An employer offering a SEP does not have to contribute a specific amount or make contribution every year.

The employer is the owner and beneficiary of the life insurance policies it purchases for participants in the SERP plan.

An employer purchases life insurance for each plan participant to informally fund the plan. The employer policies are purchased, owned, and payable to the employer.

Which of the following statements is CORRECT regarding governmental Section 457(b) plans?

An employer sponsoring a Section 457 plan does not pay federal income taxes, therefore deductibility is not an issue.

How many life insurance policies are needed to fund an entity purchase buy-sell agreement in a partnership with 5 equal partners?

An entity purchase buy-sell agreement requires 5 life insurance policies. The partnership purchases one policy insuring each of the 5 partners.

Which of the following plans is designed to provide benefits only for executives whose annual projected qualified plan benefits are limited under the dollar limits of IRC Section 415?

An excess benefit plan makes up the difference between the qualified plan benefits top executives are allowed under IRC Section 415 and the benefit permitted for rank and file employees. In other words, highly compensated employees receive the difference between the amounts payable under their qualified plan and the amount they would have received if there were no benefit limitations under IRC Section 415.

Each of the following statements regarding an in-service partial plan distribution made this year from a qualified plan is correct EXCEPT:

An in-service distribution may be transferred to an eligible retirement plan by means of a direct transfer rollover.

Which of the following is CORRECT regarding an unfunded nonqualified plan?

An unfunded arrangement is a nonqualified plan in which the employee has only the employer's "mere promise to pay" the deferred compensation benefits in the future, and the promise is not secured in any way.

Any distribution from an eligible retirement plan is eligible for rollover, except which of the following? (Select all that apply)

Any distribution from an eligible retirement plan is eligible for rollover, except a required minimum distribution, a distribution that is one of a series of substantially equal periodic payments payable for a period of ten years or more or for the life or life expectancy of the employee or the employee and a designated beneficiary, or a hardship distribution.

Profit sharing plans can use 100% of the employer contribution to purchase insurance of any type after it has been in the plan two years?

Any employer contribution that has been in the profit-sharing plan for at least two years can be used up to 100% for insurance purchases of any type as long as the plan specifies that the insurance will be purchased only with such funds.

Which of the following entities is NOT eligible to sponsor a Section 457 nonqualified deferred compensation plan?

Any organization exempt from federal income tax, except for a church or synagogue or an organization controlled by a church or synagogue.

Bill, a fiduciary of the CAP pension plan, took $10,000 of the pension plan and invested it in his own account. The $10,000 grew to $15,000 and then Bill's breach was discovered. What will Bill be responsible for?

Any person who is a fiduciary with respect to a plan who breaches any of the responsibilities, obligations, or duties imposed upon fiduciaries shall be personally liable to make good to such plan any losses to the plan resulting from each breach. They will have to restore to such plan any profits of such fiduciary, which have been made through use of assets of the plan by the fiduciary.

AAA corporation has a stock-redemption plan and is going to have to claim bankruptcy. Creditors can get access to the cash value in the stock-redemption plan. State True or False.

Any policy cash values, and death proceeds are, therefore, subject to attachment by the creditors of the corporation because the policy values are general corporate assets.

For participants to borrow from a plan, the plan must specifically permit such loans. Loan provisions are most common in defined contribution plans, particularly profit-sharing plans. From which of the following plans are loans permitted? (Select all that apply)

Any type of qualified plan or Section 403(b) tax-deferred annuity plan may permit loans. However, if the plan is subject to ERISA, loans from Section 403(b) tax-deferred annuity plans are subject to the prohibited transaction rules and penalties. Section 401(k) plans are qualified plans and therefore allow loans. There are considerable administrative difficulties connected with loans from defined benefit plans because of the actuarial approach to plan funding. Loans from IRAs and SEPs are not permitted.

Jenny needs 40 credits to qualify for benefits of Social Security. Her annual income is $80,000. Pick out the statements that are true regarding Jenny's credits in 2021. (Select all that apply)

As Jenny works and pay taxes, she earns Social Security credits. In the year 2021, she will earn one credit of each $1,470 of earnings. However, Jenny must work for at least 10 years to qualify for benefits because she can get only a maximum of four credits per year. Although her income is higher, it will not increase her eventual Social Security benefits. The amount of income she earns will increase her benefits up to just the maximum credits available.

Next month Sally Sharp will be quitting her job with a state government agency, Sweet60, which cares for senior citizens. She will be joining a public school as a teacher. She will be receiving the distribution from the Section 457 plan of Sweet60 upon severance of employment. If she wants to avoid paying tax on the distribution this year, what are the options available to her? (Select all that apply)

As an employee of a government agency, Sally Sharp would have to include the distributions in income when they are paid. However, this will result in it being taxed in the current year. To avoid paying tax she could directly roll over or make a direct transfer of the amount to other permitted plans or accounts. She can also use the amount to repay previously refunded contributions.

Smith files an FSA election with his employer to reduce his salary by $1,500, which was placed into his FSA benefit book account. At the end of the year, $600 remains in his account. What can Smith do with the amount?

At the end of the year, if expenses are less than predicted, it is forfeited. Only $550 can be carried over to the next year. The amount forfeited reverts to the employer as it means that the compensation is not to be paid.

Each of the following may help improve cash flow during retirement EXCEPT:

Being covered by Medicare and therefore not having out-of-pocket healthcare expenses is not a correct statement. Medicare requires certain deductibles and co-pays.

Blue Cross plans are used for doctors' bills, and Blue Shield plans used for hospital bills. (Select True or False)

Blue Cross plans are used for hospital bills, and Blue Shield plans used for doctors' bills.

Elsa was born in 1961 and is fully insured under Social Security. She plans on retiring from full-time employment and claiming her Social Security retirement benefit at age 62. Her current full retirement primary insurance amount (PIA) is projected to be $2,000 per month. If Elsa works part-time until age 67 and earns $2,000 per month, what is the benefit withholding formula that would apply due to her earnings in the year she attains age 65?

Born in 1961, Elsa's full retirement age under Social Security is 67. Claiming Social Security retirement benefits at age 62 and continuing to have earned income would result in benefits being withheld of $1 will be withheld for every $2 her earned income exceeds $19,560 (2022).

Elsa was born in 1961 and is fully insured under Social Security. She plans on retiring and claiming her Social Security retirement benefit at age 62. What percentage of her primary insurance amount (PIA) will Elsa receive?

Born in 1961, Elsa's full retirement age under Social Security is 67. Claiming Social Security retirement benefits at age 62 will result in a reduction of her benefit of 30%. She is retiring 60 months prior to her full retirement age. The reduction is calculated as follows: (5/9ths of 1% x 36 months = 20%) + (5/12ths of 1% x 24 months = 10%) = 30%

Elsa was born in 1961 and is fully insured under Social Security. She plans on retiring and claiming her Social Security retirement benefit at age 70. What percentage of her primary insurance amount (PIA) will Elsa receive?

Born in 1961, Elsa's full retirement age under Social Security is 67. Claiming Social Security retirement benefits at age 70 will result in an increase of her benefit of 24%. Delaying Social Security retirement benefits beyond one's full retirement age increases the benefit by 8% per year.

Why are cafeteria plans popular among employees and employers? (Select all that apply)

Cafeteria plans allow employees to choose the form of employee benefits. These plans must include a cash option, wherein employees have an option to receive cash in lieu of noncash benefits of equal value. Cafeteria plans also help control the cost of the benefit package to the employer, as the benefits that are not taken by the employees reduce the employer's provisionary costs.

A plan formula provides 40% of final average compensation with an offset. The lowest paid employee must receive at least what percentage of the final average compensation from the plan?

Code and regulations provide limits on the extent of an offset for Social Security. In particular, the rules provide that no more than half of the benefit provided under the formula without the offset may be taken away by an offset. In this case, the lowest paid employee must receive at least 20% of the final average compensation from the plan (0.40 x 0.50 = 0.20).

A law firm is made up of two partners who will earn $500,000 each this year, a secretary who will earn $40,000 and several law clerks who earn from $25,000 to $35,000 each per year. The partners and the secretary started out the firm over seven years ago. Law clerks serve for a year or two and then move on. The firm has a SEP-IRA plan. Which individuals must receive a contribution to their SEP-IRA account for this year?

Contributions do not need to be made to employees until their third year of service.

Assets of which type of annuity are NOT a part of the insurance company's general account?

Contributions made to a variable annuity are put into a separate account that is not part of the insurance company's general account.

Generally, contributions to an Coverdell ESA must be made on or before the beneficiary attains what age?

Contributions must be made on or before the date on which the beneficiary attains the age of 18.

Brad is a retirement planning analyst who wishes to determine the benefits of a Section 403(b) tax annuity plan. All of the following are benefits of a Section 403(b) tax-deferred annuity plan, except:

Contributions to a 403(b) tax deferred annuity plan may or may not be currently taxable. It depends on whether the contributions are traditional or Roth. The plan account balances can accumulate tax-free (and may even be distributed under certain circumstances tax free). Lump sums do not qualify for any special averaging.

Ted, age 45, is employed by a municipality that sponsors a government Section 457 plan with a normal retirement age of 65. His monthly salary is $7,000. Ted also works part-time for a for-profit company and earns $25,000 annually. The company sponsors a Section 401(k) plan. What is the combined maximum amount Ted can defer into the Section 457 plan and the Section 401(k) for 2022?

Contributions to a government Section 457 plan are not aggregated with contributions to another plan in which the taxpayer participates. Ted can contribute $20,500 to the Section 457 plan and $20,500 to the Section 401(k) plan.

Participation in which of the following retirement plans may affect the deductibility of an IRA? Click all the apply.

Current law imposes income limitation on the deductibility of traditional IRA contributions for those persons who are "active participants" in an employer retirement plan that is tax-favored. This includes a qualified retirement plan, simplified employee pension (SEP), Section 403(b) tax-deferred annuity plan or SIMPLE IRA.

Which of the following may decrease the necessary annual savings required to meet a retirement income funding goal for a given "in today's dollars" annual retirement income amount?

Decreased inflation assumptions may decrease the necessary annual savings required because the assumed retirement income goal (in today's dollars) will decrease due to lower inflation adjustments.

A defined benefit plan must provide a married participant with which of the following as the automatic form of benefit?

Defined benefit plans must provide a married participant with a joint and survivor annuity as the automatic form of benefit.

Which of the following are distribution options provided by defined contribution plans? (Select all that apply)

Defined contribution plans provide a lump sum benefit at retirement or termination of employment or annuity over the retirement years. Defined contribution plans often also allow the option of taking out nonannuity distributions over the retirement years as they are needed. However, these annuity distributions cannot be made before retirement.

Benefits for the following will not be paid even if the definition of disability is met EXCEPT?

Disability plans usually have specific exclusions under which benefits are not paid even if they meet the definitions of disability. All of the above are excluded from payment even if they comply with disability definition, except for medical expenses incurred for treatment of an injury that caused disability. In this case disability benefits are paid due to injury.

Employee discounts on merchandise are an advantage to the employer because they can still make a profit on the items sold or at least recover its cost but does not have to bear the full cost of marketing and selling the items to the public. State True or False.

Discounts on merchandise are almost as valuable as cash to employees but are very inexpensive for the employer because the employer can still make a profit on the items sold or at least recover its cost, but does not have to bear the full cost of marketing and selling the items to the public.

Benefit payments from a traditional IRA must begin by:

Distributions must begin by April 1 of the year after the year in which age 72 is attained. If the first RMD is deferred until April 1 of the year following the year in which age 72 is attained, the participant will be required to make two RMDs that year; one distribution by April 1 and the second by December 31.

What is the tax treatment of a distribution to a beneficiary of a seven-year-old Roth-IRA following the owner's death?

Distributions to beneficiaries after the owner's death are tax-free to the recipients, but they lose their character as Roth IRAs when distributed. Had the Roth-IRA been established for less than five years, any gain would be taxable at the owner's death.

In a qualified plan, such as a Section 401(k) plan, under which a participant has investment allocation control over the assets in their account, what is the minimum number of investment alternatives that must be offered according to ERISA regulations?

ERISA dictates that the account holder be given control over the assets in his or her account and provide at least three investment alternatives.

Shareholders interests in a buy and sell agreement are determined when a shareholder passes away. State True or False.

Each shareholder's interest is valued at the time the agreement is drafted, and it should be revalued periodically, and the agreement amended to incorporate the new values.

Early distributions from which retirement plan will attract a penalty of 25% during the first two years of participation in the plan?

Early distributions from qualified plans, Section 403(b) tax-deferred annuity plans, IRAs and SEPs are subject to a penalty of 10% of the taxable portion of the distribution. In the case of SIMPLE IRAs, the penalty is increased to 25% during the first two years of participation.

Earned income comprises which of the following? (Select all that apply)

Earned income refers to income from employment or self-employment. Investment income is not counted as earned income. Taxable alimony payments are considered earned income.

A spouse waiver of the survivorship annuity benefit must meet all of the following requirements EXCEPT:

Electing out of the preretirement survivorship benefit will generally increase the participant's benefit after retirement.

Which of the following are advantages of traditional IRAs? (Select all that apply)

Eligible individuals can contribute up to the maximum annual contribution amount to a traditional IRA, and the amount may be deducted from their current taxable income. Income earned from the assets in a traditional IRA is not taxed until it is withdrawn (subject to the 10% early withdrawal penalty).

Harry Grisham is 45 years old. He joins a company that provides a SEP to its employees. Which of the following statements is true regarding Harry and the SEP plan? (Select all that apply)

Employees must not rely upon a SEP to provide them with an adequate retirement benefit. Benefits are significant only if the employer makes substantial, regular contributions to the SEP. But such regular contributions are not a requirement for a SEP, so it may not provide significant benefits.

Employees that contribute after-tax money into their account will have to pay federal income taxes on those contributions when the money is withdrawn. State True or False.

Employees that make after-tax contributions can receive these amounts free of federal income taxes, although the order in which they are recovered for tax purposes depends on the kind of distribution.

Which of the following are characteristics of employees who value immediate cash? Click all that apply.

Employees who value immediate cash are usually younger employees who do not expect to stay with the employer long. In addition, they are often lower-paid employees as well.

Health Savings Account: Medical Savings Account Health Reimbursement Arrangement

Employer contributions are fully deductible and any contributions made on behalf of the employee are fully vested and non-forfeitable. A self-employed individual may contribute if they maintain a high-deductible health plan. A self-employed individual is not eligible to contribute.

Under current qualified plan regulations, what is the longest period over which a defined contribution plan may defer 100% vesting of employer matching contributions?

Employer matching contributions are a feature of a defined contribution plan, namely, a Section 401(k) plan. The longest period over which a defined contribution plan may defer 100% vesting of employer matching contributions is 6 years.

Which plans are either exempt from ERISA's reporting and disclosure requirements or subject to reduced ERISA reporting and disclosure requirements? (Select all that apply)

Employer-facilitated IRAs, simplified employee pensions (SEPs), SIMPLEs, and Section 403(b) TDA plans are, in some cases, either exempt from ERISA's reporting and disclosure requirements or subject to reduced ERISA reporting and disclosure requirements. Code Section 125 plans refer to cafeteria plans, which are not included in the ERISA exempt list

A Social Security number is a mandatory requirement for which of the following? (Select all that apply)

Employers, financial institutions that pay interest and the Internal Revenue Service (IRS) require that an individual present his or her Social Security number. It is not possible to pay into the Social Security program and to collect Social Security benefits without a Social Security number. Many other businesses and government agencies also use the Social Security number for recordkeeping purposes.

Ernesto and Maria, both age 49, are self-employed professionals with combined MAGI of $175,000. Maria maintains a SEP IRA and contributes the maximum allowable each year. What combined amount may Ernesto and Maria deduct this year if maximum contributions are made to traditional IRAs?

Ernesto and Maria may each contribute $6,000 (2022) to an IRA this year but only $6,000 may be deducted. Maria is an active participant in a retirement plan and their MAGI exceeds the deduction phase-out threshold, therefore, no deduction for her contribution is available. Ernesto's contribution is fully deductible because their MAGI is below the applicable deduction phaseout.

Ernesto and Maria, both age 49, are self-employed professionals with combined MAGI of $175,000. Maria maintains a SEP IRA and contributes the maximum allowable each year. What combined amount may Ernesto and Maria contribute to traditional IRAs this year?

Ernesto and Maria may each contribute $6,000 (2022) to an IRA this year. Although Maria is an active participant in a retirement plan, she is not prohibited from contributing to an IRA.

Only those people who paid into Medicare are eligible to secure Medicare Part A.

Even if a person has not paid into the system, a person can enroll in Medicare Part A by paying the required premium.

Which of the following is NOT listed as a qualified plan prohibited transaction by a fiduciary in the Internal Revenue Code (IRC)?If ordinary (whole life) life insurance is used in a defined benefit plan, what is the maximum death benefit the life insurance may provide without violating the "incidental" test for life insurance in a qualified plan?

Failing to diversify the investments of the plan to minimize the risk of large losses may be a breach of fiduciary duty by a plan fiduciary but is not listed as a prohibited transaction in the IRC.

Contributions cannot be made even if there are no current or accumulated profits in a Profit Sharing Plan.

False. Contributions can be made even if there are no current or accumulated profits. Even a nonprofit organization can have a qualified Profit Sharing Plan.

SEPs must be adopted in the year in which they are to be effective.

False. Qualified plans must be adopted before the end of the year in which they are to be effective. But, on the other hand, SEPs can be adopted as late as the tax return filing date, including extensions, for the year in which they are to be effective.

Which of the following methods may NOT be used to finance a group-term carve-out arrangement?

Financing a group-term carve-out arrangement requires the implementation of methods such as bonus plans, split-dollar plans, and death benefit plans. In order for carve-out programs to work as intended, they need to be carefully designed to avoid Section 79 status. This can be achieved if the coverage involves either/and the split-dollar, bonus, or death benefit only plans.

A defined benefit plan typically provides an unmarried participant with which of the following as the automatic form of benefit?

For an unmarried participant, a defined benefit plan's automatic form of benefit is usually a life annuity, which is typically a series of monthly payments to the participant for life, with no further payments after the participant's death.

For eligibility purposes, a year of service is defined as which of the following?

For eligibility purposes, a year of service means a 12-month period during which the employee has at least 1,000 hours of service.

Assume that Mr. And Mrs. Stevens are both participants in qualified retirement plans and that their combined modified adjusted gross income (MAGI) for 2021 is $180,000. Which of the following statements is true?

For married filing jointly taxpayer who are both participants in an employer-sponsored retirement plan, the deduction for traditional IRA contributions is fully phased out at MAGI of $125,000 (2021). However, they can make nondeductible contributions to the traditional IRA within limits.

What is the contribution deadline for a Roth IRA?

For most individuals or married couples, the contribution cutoff date is April 15th. However, since earnings on a Roth IRA account accumulate tax-free, taxpayers may want to make contributions as early as possible in the tax year.

Special credit is given to people who delay retirement beyond their full retirement age. For workers reaching full retirement age in 2022, the delayed credit rate is 8% per year up to age 70.

For people reaching full retirement age in 2022. The rate is 8% per year.

For qualified plans, how frequently must an employer provide an individual benefit statement?

For qualified plans, employers are required to provide an individual benefit statement at least once annually. This statement is very helpful for retirement planning because it provides detailed account information.

Which of the following is the annual financial reporting form for a qualified plan?

Form 5500 is required as a qualified plan's annual financial report and must be filed with the IRS each year by the end of the seventh month after the plan year ends.

From an executive's perspective, which of the following is the principal disadvantage inherent to a nonqualified plan?

From the executive's point of view, the principal problem is lack of security as a result of depending only on the employer's unsecured promise to pay. In addition, most of the protections of federal tax and labor law (ERISA) that apply to qualified plans, for example, the vesting, fiduciary, and funding requirements, are not applicable to the typical nonqualified plan.

Which of the following statements is NOT correct regarding "fully insured" qualified pension plans?

Fully insured defined benefit pension plans were once very common but the high interest rates of the late 1970s lured many pension investors away from traditional insured pension products.

Which of the following is not correct regarding stock appreciation rights?

Generally, no actual shares are set aside, nor are shares of stock necessarily actually distributed. The value of employer stock simply is the measure by which the benefits are valued.

If an employer provides and pays the premiums for long-term care insurance as an employee benefit, generally, which of the following statements correctly describes the tax treatment of the premiums and benefits paid under the policy?

Generally, premium costs are deductible to the employer, and premiums and benefits are nontaxable to the employee or beneficiary.

Grandfathered rules are applied to taxation of plan participants who attained age 50 before which date?

Grandfathered rules related to after-tax contributions and 10-year averaging are applied to taxation of plan participants who attained age 50 before January 1, 1986.

Which type of HMO plan involves medical groups but does not directly employ individual doctors and other providers?

HMOs are basically organized in one of three ways - staff model, group practice or medical group model, and individual practice association. In the group practice model, a contract is signed between the HMO and the medical group or groups that would provide the services to subscribers. However, while the HMO does not directly employ the individual doctors and other providers, subscribers must use only the services of those employed by the HMO under the contract.

Dan, age 56, wants to contribute the maximum allowable amount to his IRA account for 2021. His MAGI is $167,000 and he actively participates in his 401(k) plan. What amount can he contribute?

He can contribute $7,000, the $6,000 regular contribution and an additional $1,000 as a "catch-up" contribution as he age 50 or older. He may not be able to deduct his contribution but he can make it.

Following 15 years of service, employees of which of these employers may be eligible for the special "15 years of service" catch-up? (Select all that apply)

Health care insurance agencies, law firms and health maintenance organizations are not tax-exempt organizations, so they cannot install a Section 403 (b) plan.

Which type of plan or contract is not considered a postpaid-type health plan?

Health insurance plans can be of two types - prepaid or postpaid. The principal form of prepaid plan is the health maintenance organization (HMO), where the health care provider is paid in advance.

An excess benefit plan makes up the difference between the percentage pay that top executives are allowed under Section 415 and that which rank, and file employees are allowed. State True or False.

Highly compensated employees receive the difference between the amounts payable under their qualified plan and the amount they would have received if there were no benefit limitations under Code Section 415.

Which employment practices and benefits have been declared exempt from the ERISA reporting and disclosure requirements? (Select all that apply)

Holiday premiums Holiday gifts Recreational facilities in employer's premises Compensation during sabbatical leave

When is an IRA used? (Select all that apply)

IRAs are used: for long-term accumulation, to defer taxes on investment income, as an alternative to a qualified pension, and to shelter earned income from taxation.

Which of the following is NOT an element in the definition of a "qualified long-term care insurance contract" under Code Section 7702B?

IRC Section 4980B provides that the COBRA coverage requirements do not apply to coverage under qualified long-term care services. Qualified long-term care services means only specified services provided to chronically ill persons as defined in Code Section 7702B. So all of the above are part of the definition of qualified long-term care services except the COBRA continuation coverage requirements.

An employer's deduction for contributions to a welfare benefit fund that is not part of a 10-or-more employer plan is generally limited to which of the following?

If a VEBA is not part of the 10-or-more-employer plan and does not qualify under Code Section 419 A(f)(6), then deductible contributions are severely limited. But the VEBA income is set aside in excess of the Code Section 419A limits. So if the contributions are deductible under provisions of the Code in absence of 419 and 419A, they are done under the deduction acceleration limits of the Code Sections 419 and 419A. Thus the employer's deduction for the taxable year is limited to the qualified cost of the funds.

In what circumstances would a child (under age 18 and unmarried) receive Social Security benefits? Click all that apply.

If a parent passes away, is receiving retirement benefits, or is receiving disability benefits, a child may receive Social Security benefits, depending on their circumstances. These is a cap to the total family benefits that can be received. Regardless of when a worker wants to receive retirement benefits, a worker needs to be at least 62 years old to start receiving any retirement benefits.

Angio Corporation produces and sells equipment to hospitals and doctors for use during heart surgeries. Angio employs 12 salespeople and 26 office staff. All these individuals have been with the company for more than one year and are over age 21. Angio does not want the qualified plan to cover the sales staff. Eleven of the twelve salespeople are highly compensated and six of the office staff are highly compensated. Will the plan pass the Safe Harbor Test?

If a total of 17 of the employees are highly compensated, then 21 are non-highly compensated. Therefore, the plan would need to cover at least 15 non-highly compensated employees to pass (21 x 0.70 = 14.7). It will cover 20 non-highly compensated employees who work in the office and, therefore, passes the Safe Harbor Test.

Jack purchases an annuity with a period certain of 10 years. Jack passes away 5 years into the 10-year period. His beneficiaries will receive payments for the rest of their lives. State True or False.

If one dies before the end of the "certain period", which is generally either 10 or 20 years, payments will continue to their beneficiary until the end of that period.

If a qualified plan participant has reached age 72 and commenced required minimum distributions (RMDs), what is the penalty if less than the RMD amount is distributed?

If the annual distribution is less than the minimum amount required, there is a penalty of 50% of the amount that should have been distributed but was not distributed.

If the direct rollover method is not chosen in the case of a distribution from a qualified plan, the distribution is subject to mandatory withholding. What is the percentage of the mandatory withholding?

If the direct rollover method is not chosen in the case of a distribution from a qualified plan, Section 403(b) plan or eligible Section 457 governmental plan, the distribution is subject to mandatory withholding at 20%. Distributions from a rollover IRA and employee stock option plan cannot be rolled over.

Larry, age 50, has a vested account balance in a Section 401(k) plan of $300,000. Larry has accepted a new job and is taking a lump-sum distribution from his Section 401(k) plan and has notified his plan administrator he intends to rollover the distribution within 60 days. What amount will Larry receive in the lump-sum distribution?

If the direct transfer method is not chosen in the case of a distribution from a qualified plan, Section 403(b) plan, or eligible Section 457 governmental plan, the distribution is subject to mandatory withholding at 20%. Larry will receive $240,000 and $60,000 with be withheld for federal income taxes.

If the employee is a controlling shareholder, where should the incident of ownership be attributed?

If the employee is a controlling shareholder, more than 50% in the employer corporation, the corporation's incidents of ownership in the policy will be attributed to the majority shareholder

What are the three common approaches in which life insurance can be used in defined benefit plans? (Select all that apply)

In a combination plan, retirement benefits are funded with a combination of whole life policies and separate assets in a separate trust fund called the "side fund" or "conversion fund". In the envelope funding approach, insurance policies are simply considered as plan assets like any other assets. A fully insured pension plan is one that is funded exclusively by life insurance or annuity contracts. There is no trusteed side fund.

What is the maximum percentage of qualified plan contributions that may be allocated to ordinary (whole life) life insurance on behalf of a participant in a defined contribution plan to comply with the "incidental" regulations for life insurance in a qualified plan?

In a defined contribution plan, no more than 50% of contributions on behalf of a participant may be allocated to ordinary life insurance.

In a defined contribution plan, there is one main account where all participant money is invested. State True or False.

In a defined contribution plan, the employer establishes and maintains an individual account for each plan participant. When the participant becomes eligible to receive benefit payments, the benefit is based on the total amount in the participant's account.

In which type of partnership is each partner fully liable and involved in the management of the firm?

In a general partnership each partner is actively involved in the management of the firm and is fully liable for partnership obligations.

In a normal year, William Fitzgerald works for Jeremiah Hansen as an accountant. His annual gross salary is $80,000. What is the total percentage of Fitzgerald's gross salary that must be paid as tax to Social Security and Medicare?

In a normal year, the employee and the employer each must pay 7.65% of the employee's gross salary (6.2% for Social Security and 1.45% for Medicare), up to $142,800. Therefore the total contribution made is equal to 15.3% of Fitzgerald's gross salary.

Which type of PPO plan allows patients to self-refer themselves to specialists within the network?

In an Open Panel plan, the patient can see different primary care providers and refer himself or herself to a specialist within the network.

In what ways can life insurance in all defined contribution plans be provided to employees? (Select all that apply)

In defined contribution plans, a part of each participant's account is used to purchase insurance on the participant's life. This plan can provide (a) that insurance purchases are voluntary by participants (using a directed account or earmarking provision), (b) that the insurance is provided automatically as a plan benefit, or (c) that the insurance is provided at the plan administrator's option (on a nondiscriminatory basis).

When must a qualified plan administrator provide an individual accrued benefit statement to a plan participant who makes a request for such statement?

Individual Accrued Benefit Statement can be requested by a plan participant under the plan, and the plan administrator must provide it within 30 days.

Which of the following statements are true about VEBAs? Click all that apply.

Installing and administering a WBT or VEBA is complex and costly. A reversion of assets to the employer is effectively prohibited. Smaller employers will find these plans feasible only if they use a vendor of packaged plans provided to groups of employers. While the funding of such plans through a Section 419A(f)(6) plan is theoretically sound, there is a possibility of an IRS attack or changes in the Code affecting these programs.

Insurance outside the plan is paid for entirely by after-tax dollars, so the person is benefitting from tax deductibility. State True or False.

Insurance outside the plan paid for entirely with after-tax dollars, so there is no tax deferral.

Which of the following is NOT a way retirement savings may be enhanced through tax benefits?

Interest earned on corporate bonds is taxable.

Steve retired at 55 and his only source of income is from his investment portfolio. Steve's portfolio produced $40,000 of interest and capital gains. How many credits of coverage did Steve earn toward Social Security.

Investment earnings are not subject to Social Security tax and do not receive credits. 1?

Dave Miller is 75 and is part of his employer sponsored retirement plan. Dave does not have to take minimum distributions from his work plan or any other retirement plan that is subject to the minimum distribution rules.

It is true that Dave does not have to take minimum distributions from the plan of his current employer, but he is required to take distributions from the plans of any prior employers or from any IRA accounts that he has.

Jerome, age 40, was divorced in 2020. He currently receives $3,000 per month in alimony and earns $5,000 annually working a part-time job with no benefits on weekends. What is the maximum amount Jerome can contribute to an IRA this year?

Jerome can contribute $5,000 (2022) to an IRA. Because his divorce was finalized after January 1, 2019, the alimony he receives is not considered compensation for IRA contribution purposes. He may contribute the lesser of $6,000 or 100% of his compensation income.

Joe, age 40, was divorced in 2018. He currently receives $3,000 per month alimony and earns $5,000 annually working a part-time job with no benefits on weekends. What is the maximum amount Joe can contribute to an IRA this year?

Joe can contribute $6,000 (2022) to an IRA. Because his divorce was finalized prior to 2019 the alimony he receives is considered compensation for IRA contribution purposes. The maximum IRA contribution for an individual younger than 50 is $6,000.

Jolene, age 40, was divorced in 2018. She currently receives $3,000 per month alimony and earns $5,000 annually working a part-time job with no benefits on weekends. What is the maximum amount Jolene can contribute equally to a traditional IRA and a Roth IRA this year?

Jolene can contribute $3,000 (2022) equally to a traditional IRA and a Roth IRA. Because her divorce was finalized prior to 2019 the alimony she receives is considered compensation for IRA contribution purposes. IRA contributions must be aggregated for purposes of applying the annual maximum. Jolene's alimony ($36,000, annually) + earned income ($5,000) = $41,000 of total income. Therefore, she can fund a total of $6,000 to the IRAs, $3,000 each if evenly split.

Which of the following is NOT a purpose for an employer to purchase key employee life insurance?

Key employee life insurance is not used to fund a cross-purchase buy-sell agreement because key employee insurance is owned by the company and a cross-purchase buy-sell agreement is between owners, not between the company and the employee's estate.

In 2022, Kristi earned $100,000 as a self-employed photographer. How much must she pay in self-employment tax for the year?

Kristi will pay $14,130 in SE tax for 2022. 100,000 x 0.9235 = 92,350 92,350 x 0.153 = 14,130

Larry, age 50, has a vested account balance in a Section 401(k) plan of $300,000. Larry and his spouse are in the process of finalizing a divorce and 50% of Larry's Section 401(k) plan balance will be transferred to his soon-to-be former spouse under a Qualified Domestic Relations Order (QDRO). If Larry is in a 22% marginal income tax bracket, what is the total income tax and penalty will Larry owe on the distribution?

Larry is not subject to income tax or penalty for funds distributed under a QDRO. If not rolled over, the recipient is subject to regular income tax on the distribution but distributions under a QDRO are exempt from the 10% early withdrawal penalty tax.

Most legal services plans are not designed to cover catastrophic legal expenses, such as a criminal trial. State True or False.

Legal expenses such as the cost of a criminal trial can be the kind of catastrophic expense that is best provided through an insurance-type or group benefit program such as a legal services plan.

Which of the following are advantages of having life insurance in retirement plans? (Select all that apply)

Life insurance provides one of the safest available investments for a qualified plan. In addition, the use of appropriate life insurance products for funding a qualified plan can provide extremely predictable plan costs for the employer. Also, the "pure insurance" portion of a qualified plan death benefit is not subject to income tax. This makes it an effective means of transferring wealth.

Life with period-certain annuities does not provide payments for the life of the annuitant, but for a specified period of time, usually 10 to 20 years, in which of the following circumstances? (Select all that apply)

Life with period-certain annuities provides payments for a specified period of time, usually 10 to 20 years, even if the participant, or the participant and spouse, both die before the end of that period. Thus, the life with period-certain annuity makes it certain that periodic benefits will continue for the participant's heirs even if the participant and spouse die early.

Lisa, age 40, has an eligible unforeseen financial emergency and must withdraw $30,000 from her governmental Section 457(b) plan to pay emergency expenses. If Lisa is in a 22% federal marginal income tax bracket, what is the total she will pay in income taxes and penalties?

Lisa will pay $6,600 in federal income tax. Early distributions from a governmental Section 457(b) plan are not subject to penalty.

Lori covered under a coinsurance provision and is responsible for 20% of covered expenses. She has a deductible of $250. Lori incurs expenses of $2,000 during the year. How much will she have to pay and how much will the company pay?

Lori will pay $600 ($2,000-$250=$1,750x20%=$350($250+$350=$600)) and the company will pay $1,400($2,000-$600).

Lynn has net earnings from self-employment of $120,000 annually with self-employment tax of $16,596. If she maintains a profit-sharing plan with 25% annual contributions what is the maximum contribution she can make to her account this year?

Lynn may contribute $22,340 to her profit-sharing account this year. $120,000 - (50% x $16,596) = $111,702 $111,702 x 0.20 (0.25 ÷ 1.25) = $22,340

Under the Consolidated Omnibus Budget Reconciliation Act of 1985 (COBRA), an employer must provide an option to continue health insurance coverage for an employee whose termination from employment is due to disability for what period of time?

Many employers provide benefit plans of continuing insurance coverage for employees and their dependents for some time after the termination of employment under the COBRA rule. COBRA requires that if the termination of employment is due to disability, then the coverage should continue for a period of 29 months. If the termination is due to death of the employee, divorce or separation, or bankruptcy, then the continuation period is 36 months, and in all other cases, except misconduct, the time period is 18 months.

Maria has net earnings from self-employment of $100,000 this year. Her self-employment tax is $14,130. What is the maximum amount she may contribute to a SEP on her behalf this year?

Maria may contribute up to $18,587 to a SEP plan on her behalf this year. Net earnings from self-employment ($100,000) minus ½ of the SE tax ($7,065) = $92,935 $92,935 x 0.20 (0.25/1.25) = $18,587

Hans, age 65, has just retired and started collecting his Social Security retirement benefits. His wife, Martha, is 57 years of age and has stayed at home raising their children while Hans was the breadwinner. They have four children who are living with them at present. Andrew, age 21, is studying in college. Peter, age 19, met with an accident and is severely disabled. Sandra, age 18½, has dropped out of school while Karen, age 14, is going to school. Which of the members in Hans's family are not eligible to receive Social Security benefits based on his record? (Select all that apply)

Martha is under age 62 but she can receive benefits because she is caring for Karen who is under age 16. Peter is disabled and above 19, so he can also receive benefits. Karen is under age 18 and is eligible for benefits. Andrew is not eligible by any of these rules. Sandra is also not eligible, because although she is under 19, she is not studying full-time in any school.

Section 457 plans are subject to the minimum distributions requirements under Section 401(a)(9). State True or False.

Minimum distributions must be made under the rules of Section 401(a)(9) (distributions from IRAs must generally begin as of age 70 ½), which apply to other tax-advantaged plans as well.

At what age must a participant in a governmental Section 457(b) plan commence required minimum distributions (RMDs)?

Minimum distributions must be made under the rules of Section 401(a)(9) must generally begin as of age 72.

Match the corresponding types of plans on the left with the descriptions on the right.

Money Purchase: One of the simplest of all retirement plans. The employer must contribute each year to the plan. Target Benefit: A defined contribution plan where the participant's age at plan entry is considered when determining the contribution percentage. Profit Sharing: In these plans, employer can choose not to contribute. Defined Benefit: These plans are funded actuarially. The employer assumes the investment risk.

Each of the following statements is correct regarding a breach of fiduciary duty by a qualified plan fiduciary EXCEPT:

No fiduciary shall be liable with respect to a breach of fiduciary duty if such breach was committed before he became a fiduciary or after he ceased to be a fiduciary.

A participant exercises control over and makes an exchange in his account. The plan is liable if the exchange eventually causes a loss within the participant's account.

No person who is otherwise a fiduciary shall be liable for any loss, or by reason of any breach, which results from such participant's or beneficiary's exercise of control.

Each of the following statements regarding nonqualified plans is correct EXCEPT:

Nonqualified plans are not subject to all the ERISA rules applicable to qualified plans. This gives the employer much flexibility in plan design.

Sara Jones is 55 years of age and has taught at the state university for over 15 years. Her tax-deferred annuity plan allows her to make the maximum elective deferral permitted by law, including catch-up contributions. Sara does not participate in any other salary deferral plan. What is the maximum salary deferral she can make to the plan in the 2021 plan year?

Normally, Sara could contribute the maximum salary deferral amount for 2021 of $19,500. However, she can take advantage of the age 50 or older catch-up of $6,500 and the "15 years of service" catch-up of another $3,000 for a total of $29,000 assuming that her income from the university is at least that amount.

All qualified plans must pass one of the following tests EXCEPT:

Not all qualified plans are required to pass the 50/40 test. Only defined benefit plans are required to pass the 50/40 test and the 50/40 test must be passed in addition to the safe harbor test, the ratio percentage test, or the average benefits test.

Which of the following statements below is NOT correct regarding a voluntary employees' beneficiary association (VEBA) plan?

Once contributed to the plan, funds in a VEBA cannot revert to the employer.

Each of the following statements regarding cafeteria plans is correct EXCEPT:

One of the attractive features of a cafeteria plan is that the benefits offered can be customized to best suit the various needs of a specific company's employees; benefits are not standardized.

Which of the following is an absolute requirement for a qualified distribution from a Roth IRA?

Only a 5-year holding period is an absolute requirement for a qualified distribution. There cannot be a qualified distribution without meeting the 5-year holding period requirement. Death, disability, attainment of age 59.5, or first-time home purchase are possible qualifying circumstances for a qualified distribution.

Under which of the following plans is the benefit amount at retirement guaranteed?

Only defined benefit plans guarantee the benefit at retirement. Of the plans listed, only the cash balance plan is a defined benefit plan.

What are the main differences between PPOs and HMOs? Click all that apply.

PPOs typically differ from HMOs in two aspects: First, they provide benefits on a fee-for-service basis as their services are used. Fees are usually subject to a schedule that is the same for all participants in the PPO. Second, plan participants have financial incentives to use the preferred provider network. The primary care physician does not control a participant's access to specialists, as is the case in most HMO plans.

Which part of the Medicare coverage includes hospital insurance benefits?

Part A provides hospital insurance benefits.

Part of the process of implementing a 401(k) plan is to "sell" the plan to the employees.

Part of the process of implementing a 401(k) is to "sell" the plan to the employees so that all employees, including highly compensated employees, can fully benefit from the plan. If the nonhighly compensated employees are not participating, it limits the amount of participation of highly compensated employees.

Which of the following is NOT a component in applying the annual additions limit?

Participant account earnings attributed to employer contributions or employee contributions are not a component in applying the annual additions limit.

Participants may be not excluded or required to pay an extra premium on the basis of which of the following? Click all that apply.

Participants may not be excluded or required to pay an extra premium on the basis of the following: health status, medical condition (physical and mental), claims experience, receipt of health care, medical history, genetic information, evidence of insurability, or disability.

Social Security's definition of disability is very specific. What are all the requirements that a person must meet to be considered as disabled by the Social Security Administration? (Select all that apply)

Persons are considered disabled if they cannot do the work they did before. The Social Security administration must then decide that the disabled cannot adjust to other work because of their medical condition(s). Moreover, the disability must last or be expected to last for at least a year or to result in death. In contrast the dictionary defines disability as a physical or mental condition that prevents a person from leading a normal life, which is not specific and therefore can be debated from person to person.

A plan loan to an owner-employee is permitted under the same requirements that apply to those from qualified plans. State True or False.

Plan loans to an owner-employee are not prohibited transaction under the code, provided the other plan code regulations for loans are followed.

What variables make it difficult to create a financial plan that is exact? Click all that apply.

Planners and clients should not expect exactness in the financial planning targets. There are too many variables, such as future investment return rates, future tax rates and client's exact life expectancy. Nevertheless, the targets for retirement planning are near enough to try quantify plans and make sure they are systematically carried out.

Which type of benefit provides the most tax advantages to executives in prefunded welfare benefit plans? Click all that apply.

Prefunded welfare benefit plans designed primarily for executives generally provide severance pay benefits, death benefits, or a combination of the two, since these are the benefits that tend to provide the most tax advantages.

Premature Roth IRA withdrawals in excess of contributions may be subject to: (Select all that apply)

Premature Roth IRA withdrawals in excess of contributions are taxed in full and may also be subject to a 10% penalty on premature withdrawals.

Which of the following employer-provided employee benefit plans does NOT provide a death benefit or capital accumulation? (Select all that apply)

Qualified plans such as profit sharing, group-term life insurance, and nonqualified deferred compensation all provide some form of death benefits or capital accumulations in their plan. But dependant care assistance or short-term disability plans do not provide for death benefits, as they are provided only for a small period of time and do not consider benefit provision in lieu of death.

Retirement plans are required to provide the same distribution options. State True or False.

Retirement plans may be different in regards to distributions. It is important to review the summary plan document (SPD) of a plan to identify the plan's distribution options.

Which of the following best describes a revenue ruling? Click all that apply.

Revenue rulings are published by the IRS as general guidance to all taxpayers. The IRS publishes its Revenue Rulings in IRS Bulletins and is binding on IRS personnel on the issues covered in them.

Which of the following nonqualified plans typically uses generally uses a non-elective defined benefit type of formula to calculate the benefit amount?

Salary continuation generally refers to a type of non-elective nonqualified deferred compensation plan that provides a specified deferred amount payable in the future. A salary continuation formula generally uses a defined benefit type of formula to calculate the benefit amount.

Salary reductions elected after compensation is earned are ineffective as a result of the tax doctrine of

Salary reductions elected after compensation is earned are ineffective as a result of the tax doctrine of constructive receipt.

Which of the following statements is NOT correct regarding a flexible spending account (FSA)?

Salary reductions elected by employees to fund nontaxable benefits under the plan are not subject to payroll taxes resulting in savings to both the participant and the employer.

Sally, age 50, is a freelance photographer and earns $200,000 per year. She wants to open a traditional IRA as she has not yet started a retirement plan and wants to know the maximum amount she can deduct for an IRA contribution this year assuming she contributes the maximum allowable. What amount can Sally deduct?

Sally can contribute and deduct $7,000 (2022). She is not an active participant in a retirement plan, therefore, her earnings do not prohibit her from deducting the full contribution. She is eligible for a $6,000 regular contribution plus a $1,000 age 50+ catch-up contribution.

Each of the following plans is exempt from ERISA provisions EXCEPT:

Section 401(k) plans funded exclusively by employee elective deferrals are qualified retirement plans and subject to ERISA.

Which of the following employers would NOT be subject to Code Section 457's rules for nonqualified deferred compensation plans?

Section 457 applies to nonqualified compensation plans of state and local government employers and tax-exempt employers/organizations.

COBRA provides for continued coverage to all except? Click all that apply.

Self-employed individuals, independent contractors, and directors are not counted and exempted from COBRA.

Which of the following services is allowed from qualified retirement planning services?

Services may include general advice regarding the employee's and the spouse's overall plan for retirement, of which the employer's qualified plan is only a part.

Joe and Samantha wish to exclude from their income the value of certain retirement planning services provided by their employer for this year. The employer is currently maintaining their qualified retirement plan. Choose the correct options regarding qualified retirement planning services from the given options. (Select all that apply)

Services such as tax preparation, accounting, legal, or brokerage services are included in income, without regard to whether they relate to retirement planning.

Which of the following vesting schedules is NOT permitted for employer contributions to a defined benefit plan?

Seven-year cliff vesting is not permitted in any qualified plan. Three-to-five-year graded vesting or 5-year cliff vesting is permitted for a defined benefit purchase pension plan.

Juan, age 40, died recently in a car accident, leaving his spouse and three minor children. He was fully insured under Social Security. What amount will his family receive as a one-time lump-sum death benefit?

Social Security provides a one-time lump-sum death benefit of $255. The death benefit is not per beneficiary.

In retirement needs analysis, Social Security retirement benefits are adjusted for inflation to the retirement year and subtracted from the current income replacement need.

Social Security retirement benefits will automatically be adjusted for inflation so the currently projected benefit is subtracted from the current income replacement need.

To motivate individuals with a disabling condition to go back to work, Social Security will continue to pay a partial benefit to the individual for an extended period of time. State True or False.

Social work incentives provided by Social Security allow disabled persons to work for a period of time without reducing their benefits.

Which of the following is true about Employee Stock Purchase Plan requirements?

Some of the following requirements of Employee Stock Purchase Plans include: Must be exercised within a specific time period. Must be an employee of the corporation, its parent, or subsidiary. Must be approved by stockholders. The exercise price must not be less than 85% below FMV of the stock.

What are some of the problems for a business without a funded buy-sell agreement? (Select all that apply)

Some of the problems for a business without a funded buy-sell agreement are: · No set value for business · No identified buyer(s) who must buy · No established funding · No requirement for family or heirs to sell to surviving owners or employees

In a split dollar life insurance plan, two parties divide, or "split," the responsibilities and the rights to which of the following?

Split dollar life insurance is an arrangement between an employer and an employee which involves a sharing of the costs and benefits of the life insurance policy. Usually these plans involve a splitting of premiums, death benefits, and/or cash values.

Sue, age 50, has taught in the same public high school for 20 years but is participating in the school district's Section 403(b) for the first time this year. What is the maximum contribution Sue may make to the plan this year if her annual salary is $90,000?

Sue may contribute up to $30,000 (2022). Because Sue has been employed by the same school for at least 15 years and has not previously contributed the maximum amount, she is eligible for a special catch-up contribution allowance of $3,000 in addition to the regular contribution limit of $20,500 and the age 50+ catch up allowance of $6,500.

Sue, age 40, is paid a salary of $120,000 per year. Her employer sponsors a group term life insurance plan providing coverage of three times salary. If the IRC Section 79 rate for Sue's age is $0.10 per thousand, per month, what amount of Sue's taxable benefit must be recognized for this year?

Sue must recognize an economic benefit of $372: 3 x 120,000 = 360,000 360,000 - 50,000 = 310,000 310,000/1,000 = 310 310 x 0.10 x 12 = 372

Suzi, age 40, is paid a salary of $120,000 per year. Her employer sponsors a group term life insurance plan providing coverage of three times salary. If the IRC Section 79 rate for Suzi's age is $0.10 per thousand, per month, what amount of Suzi's taxable benefit must be recognized for this year if she contributes $10 per month for the coverage?

Suzi must recognize an economic benefit of $252: 3 x 120,000 = 360,000 360,000 - 50,000 = 310,000 310,000/1,000 = 310 310 x 0.10 x 12 = 372 372 - 120 = 252

Ted has purchased an annuity for $20,000 that is expected to pay him $300 per month for the rest of his life. Ted's life expectancy is 20 years. How much of each payment is taxable to Ted?

Taxable = Exclusion Ratio (Payment) = 1-(Investment in contract/Expected payout) x payment = 1-($20,000/$72,000) x $300 = 216.67

Ted, age 54, is employed by a municipality that sponsors a government Section 457 plan with a normal retirement age of 65. His monthly salary is $7,000. Ted also works part-time for a for-profit company and earns $25,000 annually. The company sponsors a Section 401(k) plan. What is the maximum amount Ted can defer into the Section 457 plan for 2021?

Ted can contribute $27,000 to the Section 457 plan for 2022 ($20,500 regular contributions plus the $6,500 age 50+ catch-up allowance).

Ted, age 62, is employed by a municipality that sponsors a government Section 457 plan with a normal retirement age of 65. His monthly salary is $7,000. Ted also works part-time for a for-profit company and earns $25,000 annually. The company sponsors a Section 401(k) plan. What is the maximum amount Ted can defer into the Section 457 plan for 2022?

Ted can contribute $41,000 to the Section 457 plan for 2022 under the "last 3 years rule" prior to the normal retirement age in the plan. The "last 3 years rule" allows deferrals up to two times the normal annual maximum. The age 50+ catch-up allowance may not be used in the same year the "last 3 years rule" catch-up is used.

Under a group disability insurance plan, which definition of disability is MOST advantageous to the insurance company?

The 'any occupation definition of disability' is most advantageous to the insurance company because it is the most difficult definition of disability for the insured to qualify for benefits.

What is the maximum possible contribution to a SEP on behalf of a participant this year?

The 2022 annual SEP contributions are limited to the lesser of 25% of compensation (capped at $305,000), not to exceed $61,000.

Which of the following imposes extensive reporting and disclosure requirements on a broad range of employee benefit plans?

The Employee Retirement Income Security Act of 1974 (ERISA) imposes extensive reporting and disclosure requirements on a broad range of employee benefit plans. According to these provisions, various forms and information must be disclosed to plan participants and/or filed with the IRS or the Department of Labor.

Regarding qualified retirement plans, which entity administers the taxation of contributions and benefits and enforces funding, participation, and vesting standards?

The IRS administers the taxation of contributions and benefits and enforces funding, participation, and vesting standards.

What are the IRS requirements for a profit-sharing plan that prevent it from being deemed terminated?

The IRS requires substantial and recurring contributions, or the plan may be deemed to have terminated. This contribution flexibility is very advantageous for small business, whose income may fluctuate substantially from year to year.

Which of the following is a SIMPLE IRA limitation?

The SIMPLE-IRA does not have a percentage limit. In 2021, the maximum dollar limit for employee deferrals is $13,500.

Who is authorized by ERISA to grant administrative exemptions from Section 406 and 407(a)?

The Secretary of Labor is authorized by ERISA to grant administrative exemptions from Section 406 and 407(a).

The Secretary of Labor may not grant an exemption under subsection 1108 unless he finds that such an exemption is which of the following? Click all that apply.

The Secretary of Labor may not grant an exemption under subsection 1108 unless he finds that such an exemption is: administratively feasible, in the interest of the plan and of its participants and beneficiaries, and protective of the rights of participants and beneficiaries of such plan.

Failure to roll over the distribution within 60 days subjects it to income taxes, even if the employee may be eligible to elect 10-year averaging. Which of the following gives the Secretary of the Treasury the right to waive the 60-day rule? (Select all that apply)

The Secretary of the Treasury may waive the 60-day rule where it would be against equity or good conscience to enforce it, including cases of disaster, casualty or other events beyond the participant's control. However, there is no legislative basis for waiving the 60-day rule for pre-2002 distributions, even where the delays were the result of erroneous advice or the inaction of third parties.

The Social Security definition of disability is defined as

The Social Security definition of disability is very specific and more restrictive than definitions of disability found in private disability insurance policies: A condition under which the individual is unable to engage in any substantial gainful activity by reason of any medically determinable physical or mental impairment which can be expected to last for a continuous period of not less than 12 months or result in death.

Which of the following is a summary of financial information from Form 5500 that must be provided to plan participants each year within nine months of the end of the plan year?

The Summary Annual Report is a summary of financial information from the Annual Report (Form 5500 series) that must be provided to plan participants each year within nine months of the end of the plan year.

Which of the following is intended to describe the major provisions of the plan to participants in simple language?

The Summary Plan Description (SPD) is intended to describe the major provisions of the plan to participants in simple language. An SPD must be furnished automatically to participants within 120 days after the plan is established, or 90 days after a new participant enters an existing plan.

Each of the following statements is correct regarding a funded nonqualified deferred compensation EXCEPT:

The amounts are deductible by the employer when the amount is includible in the employee's income, that is when the employee has constructive receipt of the funds.

The maximum annual additions limit to a defined contribution plan on behalf of a participant in 2021 is:

The annual additions limit for 2021 is the lesser of 100% of the participant's covered compensation or $58,000. Annual additions includes employee contributions, employer contributions, and reallocated forfeitures.

The below-market loan rules between an executive and his company will not apply to a compensation-related loan for any day on which the amount of all loans between the executive and his company do not exceed which amount?

The below-market loan rules between an executive and his company will not apply to a compensation-related loan for any day which the amount of all loans between the executive and his company do not exceed $10,000.

The waiver of the preretirement survivorship benefit in favor of an optional benefit has to be consented to by the nonparticipant spouse. The consent to waiver must meet which of the following requirements? (Select all that apply)

The consent of the nonparticipant spouse to waiver of the preretirement survivorship benefit in favor of an optional benefit form selected by the participant must be in writing, acknowledge the effect of the waiver and be witnessed, either by a plan representative or a notary public. There is no regulation about certification by a plan administrator.

Which of the following variables is NOT typically used in the year-to-year actuarial assumptions in the funding requirements for a defined benefit plan?

The consumer price index (CPI) is not typically a variable used in the year-to-year actuarial assumptions in the funding requirements for a defined benefit plan. Assumed compensation increases are an incorporated variable but not the CPI itself as a separate variable.

Taxable part of a plan distribution is determined by the total cost basis divided by the total payout. The cost basis includes: (Select all that apply)

The cost basis includes employee after-tax contributions, cost of life insurance reported as taxable income, employer contributions taxed to employee and plan loans included as taxable income. However, rollovers are not included in cost basis because they are not taxed at the time of being rolled over. They are generally made to defer tax until the time the funds are actually withdrawn from the plan

What are the taxation rates used to measure the value of group-term life insurance in excess of $50,000 generally referred to as?

The cost of the first $50,000 of group-term insurance is tax-free to the employees. However, for coverage above $50,000, the amount taxable to the employees, or rather the value of the group-term life insurance needs to be calculated on a monthly basis. This is done by multiplying by the Table I rates..

What is the tax consequence of the death benefit paid under a key employee life insurance policy?

The death proceeds of key employee life insurance are tax-free for regular income tax purposes when paid to the corporation.

Which of these employee groups can never be excluded from group-term life insurance that an employer maintains?

The design feature of group-term life insurance includes the exclusion criteria. When calculating the percent of employees to be covered under the group-term insurance, employees with less than three years of service, part-time/seasonal employees, and employees of the collective bargaining unit can be excluded. Employees with greater than three years service are included in the plan.

Larry and Kristi select a joint and survivor annuity with a 100 percent survivor benefit. What are the disadvantages to them of selecting a 100 percent survivor benefit versus a 50 percent survivor benefit?

The disadvantage of a joint survivor annuity with a 100 percent survivor benefit is that the initial benefit received will be less because there is a guarantee that the survivor will receive the same payout. A joint and survivor annuity with a 50 percent payout will pay a higher initial benefit due to the fact that only 50 percent of the benefit will be paid upon one of the annuitants' deaths.

The early distribution will attract a penalty if: (Select all that apply)

The early distribution penalty does not apply to distributions made on or after attainment of age 59½. Early distribution will also not attract a penalty if it is made to the plan participant's beneficiary or estate on or after the participant's death. The penalty is also not applicable if it is made to pay health insurance costs while unemployed. However, the early distribution penalty is applicable if it is made upon separation from service before attainment of age 55.

Which of the following is NOT correct regarding the tax treatment of a nonqualified plan?

The earnings of plan assets set aside in currently taxable investments to informally fund a nonqualified deferred compensation plan do not provide the employer with an offsetting tax deduction. The employer receives no tax deduction until the employee has constructive receipt of the benefits.

Which of the following statements is correct regarding the tax treatment of the economic value of pure life insurance held by a participant in a qualified plan?

The economic value of pure life insurance is taxable annually to the participant. Any premium contributed to the plan by the participant is subtracted from the taxable annual economic benefit amount.

It is the employer's sole responsibility to keep the employee's record of earnings correct with the Social Security Administration. State True or False.

The employee shares with the employer for making sure that all the earnings have been reported and they are accurate. The accuracy is important because the employee's benefits will be based on this record of lifetime earnings.

Patty is retiring this year and has a Section 401(k) account balance of $500,000, of which $200,000 is invested in employer stock. The employer basis in the stock when contributed was $50,000. If Patty takes a lump-sum sum distribution from the plan and makes a net unrealized appreciation (NUA) election, what is the tax treatment of the distribution of the employer stock?

The employer basis in the stock is taxed as ordinary income in the year of the lump-sum distribution and the gain at the time of the distribution is taxed as long-term capital gain when the stock is subsequently sold. In this example, $50,000 is taxed as ordinary income this year and $150,000 is taxed as long-term capital gain when Patty sells the stock.

Patty is retiring this year and has a Section 401(k) account balance of $500,000, of which $200,000 is invested in employer stock. The employer basis in the stock when contributed was $50,000. If Patty takes a lump-sum sum distribution from the plan and makes a net unrealized appreciation (NUA) election, what amount is taxed as long-term capital gain this year?

The employer basis in the stock is taxed as ordinary income in the year of the lump-sum distribution and the gain at the time of the distribution is taxed as long-term capital gain when the stock is subsequently sold. In this example, $50,000 is taxed as ordinary income this year and $150,000 is taxed as long-term capital gain when Patty sells the stock. The example does not state Patty sold the stock this year.

What are the advantages of a nonqualified stock option plan? (Select all that apply)

The employer has the flexibility to design the plan to suit both himself or herself and the employee. The company bears little or no out-of-pocket cost for these plans. For the employee, tax is generally not payable at the time when a stock option is granted. Taxation occurs when the option is exercised.

If an employer funds a Section 457 plan with life insurance at no current cost to an employee, who should be the sole beneficiary of the policy?

The employer must be the sole beneficiary in an employer funds a Section 457 plan with life insurance at no current cost to the employee.

Employer plans exempt from ERISA are which of the following? (Select all that apply)

The employer plans that are exempt from ERISA are plans of state, federal, or local governments or governmental organizations. Plans for churches, synagogues, or related organizations are also exempt. If plans are maintained outside the United States for aliens, then they are exempt. Unfunded excess benefit plans that are a type of nonqualified deferred compensation plan, and plans that are maintained solely to comply with workers' compensation, unemployment compensation or disability insurance laws are also exempt from ERISA.

Under an unfunded nonqualified deferred compensation plan, when does the employer receive a tax deduction for benefits provided?

The employer receives a tax deduction when employees receive benefits.

Patty is retiring this year and has a Section 401(k) account balance of $500,000, of which $200,000 is invested in employer stock. The employer basis in the stock when contributed was $50,000. If Patty dies before making the lump-sum distribution, what amount is subject to inclusion in her gross estate for estate tax purposes?

The entire value of a qualified plan death benefit is subject to inclusion in the decedent's gross estate for federal estate tax purposes.

Even if the earned income is not used for consumption, the federal income tax system imposes tax on income from savings. What are the three major exceptions when taxes can be avoided? (Select all that apply)

The federal income tax system imposes tax on income from savings even if it is not used for consumption, with only three major exceptions: deferral of tax on capital gains until realized, exclusion of gain on the sale of a personal residence, and deferral of tax and other benefits for qualified retirement plans and IRAs.

What is the total number of policies needed for a cross-purchase arrangement if there are seven employees?

The formula is n(n-1). 7(7-1) =42

The general rule for an employee's qualified plan participation eligibility is

The general rule for an employee's qualified plan participation eligibility is age 21 and 1 year of service with the business.

In a retirement needs analysis calculation, if the assumed rate of return is 6% and the assumed inflation rate is 2%, what is the inflation-adjusted rate of return?

The inflation-adjusted rate is 3.92%. [(1.06 ÷ 1.02) - 1] x 100 = 0.0392. Alternatively, the inflation-adjusted rate can be calculated: (6 - 2) ÷ 1.02 = 3.92%

Loans can be given to employees from the 401(K) plan contributions made by them. However, interest on these loans is not tax-deductible by the employee unless which of the following?

The interest for 401(K) loans is typically not deductible but an exception does exist for primary residence loan interest.

The basis of all regulation, which is treated as the highest level of authority by the U.S. Congress, is which of the following?

The law as expressed by statutes passed by the U.S. Congress is the highest level of authority and is the basis of all regulation. The court cases, rulings, and regulations are simply interpretations of the statute.

What difficulties can be avoided upon the death of a partner if members of a partnership enter into a buy-and-sell agreement? (Select all that apply)

The law provides that upon the death of a general partner, the partnership is dissolved. Dissolution of the partnership is not taxed, but the deceased partner's interest in the partnership may have to be used in paying his or her estate tax. This may result in the forced sale of assets, usually at a fraction of their normal value, and goodwill is completely lost. It usually results in them losing their very means of earning a living.

Worthwhile Inc. is a nonprofit organization that employs over 1,000 people. It adopted two Section 457 plans. Plan One provides benefits within $19,500 limits. Plan Two provides additional cash benefits. These supplemental cash benefits cease to exist upon premature retirement or unforeseen death of the employee. What problems are bound to occur while designing the forfeiture provisions of such retirement plans? (Select all that apply)

The main problems encountered in designing such plans would be to develop forfeiture provisions that are substantial enough to defer taxes and yet are acceptable to the executive. It is also quite difficult to design a bona fide, substantial forfeiture provision that extends past the executive's retirement. Section 457(f) amounts generally are taxable in full, no later than the year of the executive's retirement.

Assume that in 2021, Kate, age 35, contributes $2,000 to a traditional deductible IRA. Kate's AGI is $50,000. How much can she contribute to a Roth IRA for 2021 if the payment is made before April 15, 2022?

The maximum Roth-IRA contribution for an individual is the lesser of: the dollar limit for 2021 ($6,000) or 100% of the individual earned income. Since Kate has contributed $2,000 to a traditional deductible IRA, she can only contribute an additional $4,000($6,000-$2,000=$4,000) to a Roth-IRA for 2021.

The maximum annual deductible employer contribution to a profit-sharing plan is

The maximum annual deductible employer contribution is 25% of aggregate covered compensation.

This year, what is the maximum deductible employer contribution to a money purchase pension plan?

The maximum deductible employer contribution to a money purchase pension plan, which is a defined contribution plan, is 25% of aggregate covered compensation.

Ted, age 45, is employed by a municipality that sponsors a government Section 457 plan with a normal retirement age of 65. His monthly salary is $7,000. What is the maximum amount Ted can defer into the Section 457 plan for 2022?

The maximum deferral into a government Section 457 plan is the lesser of 100% of compensation or $20,500 if no catch-up provisions apply (2022).

Larry, age 50, has a vested account balance in a Section 401(k) plan of $300,000. He occasionally takes loans from the account to finance various purchases. Ten months ago, he borrowed $10,000 for a cruise and finished repaying the loan last month. Now, Larry wants to borrow the maximum amount from the plan to make a down payment on a new home. What is the maximum loan Larry make take from the plan today?

The maximum loan Larry may take today is $40,000. IRC Section 72(p) provides that aggregate loans from qualified plans to any individual plan participant cannot exceed the lesser of: $50,000, reduced by the excess of the highest outstanding loan balance during the preceding one-year period over the outstanding balance on the date when the loan is made, or One-half the present value of the participant's vested account balance or accrued benefit, in the case of a defined benefit plan.

If a client needs to accumulate $2,144,922 by the first day of retirement in 20 years, what level amount must the client save at the end of each month to reach the goal assuming annual inflation of 2% and an annual return of 7%?

The needs to save $4,118 at the end of each month to reach the retirement funding goal. END mode 2,144,922, FV 0, PV 20 x 12 = 240, N 7 ÷ 12 = 0.5833, I/YR Solve PMT 4,117.51 or, $4,118 (rounded)

James Stewart, age 60, dies in July 2021 before retirement. He has elected his wife Anita as his beneficiary. Anita receives a lump sum death benefit of $250,000 from a life insurance policy held by a qualified plan. The insurance contract's cash value was equivalent to $160,000 at James's death. During his lifetime, James had reported an insurance cost of $25,000. What is the nontaxable amount of the $250,000 distribution?

The nontaxable amount is the total of the participant's cost basis and the pure insurance amount. Therefore, it is calculated as follows: $250,000 (death proceeds) - $160,000 (cash value) = $90,000 (pure insurance amount) + $25,000 (cost basis) = $115,000 (nontaxable amount).

If ordinary (whole life) life insurance is used in a defined benefit plan, what is the maximum death benefit the life insurance may provide without violating the "incidental" test for life insurance in a qualified plan?

The participant's insured death benefit must be no more than 100 times the expected monthly pension benefit (100 times limit).

John has a qualified plan that has $100,000 of life insurance on his life with his wife as the named beneficiary of his qualified plan and his children as the contingent beneficiaries. Who is the owner of the life insurance policy?

The plan trustee is the owner according to The Retirement Equity Act of 1984.

If a business purchases business overhead expense insurance insuring the owner of the business, generally, which of the following statements correctly describes the tax treatment of the premiums and benefits paid under the policy?

The premiums paid by the business are tax-deductible by the business and the benefits paid under the policy are taxable to the business. While the benefits paid are taxable to the business, the benefits will be used to pay deductible business expenses, therefore, no tax liability will be incurred.

Neely participates in her retirement plan. She received a notice to elect a survivorship benefit, but never made the election. What is her automatic benefit?

The preretirement survivor annuity is an automatic benefit. If no other election is made, a preretirement survivor annuity is provided.

The primary responsibility of a qualified plan fiduciary is to

The primary responsibility of a qualified plan fiduciary is to run a plan solely in the interest of participants and beneficiaries.

Which of the following options regarding nonqualified plan vesting rules are true? (Select all that apply)

The qualified plan vesting rules apply in the case of rank-and-file employees. If the plan covers only independent contractors or a select group of management or highly compensated employees, benefits may be forfeitable in full at all times.

Which of the following are the requirements of the Code Section 4975(d)(1)? (Select all that apply)

The requirements of the Code are that loans are available to all participants on a reasonably equivalent basis and in accordance with specific provisions set forth in the plan. The loans must bear reasonable rates of interest and must be adequately secured. Loans must not be made available to highly compensated employees in an amount greater than the amounts made available to other employees.

Each type of qualified plan meets certain employer objectives better than others. Which of the following is NOT an accurate description of plan features that meet specific objectives?

The savings account feature of defined contribution plans attracts younger employers. A defined benefits plan will work well to encourage retirement. It may not meet the objective of assuring adequate income. An ESOP or profit sharing plan tries to create an incentive for employees to maximize performance.

Of the disability definitions described below, which is the strictest one or the least preferred by employees?

The strictest definition of disability is the total and permanent definition that states the disability condition under which an employee is unable to engage in any substantial gainful activity by reason of any medically determinable physical or mental impairment which can be expected to result in death or which has lasted or can be expected to last for a continuous period of not less than 12 months. Because of the nature of the defining disability, it is least favorable to the employees.

The three investment alternatives suggested in IRC Section 404(c) for qualified plans include each of the following EXCEPT:

The three investment alternatives suggested in IRC Section 404(c) for qualified plans include stocks, bonds, and cash equivalents.

Which of the following is true regarding the traditional income tax consequences of a split-dollar arrangement?

The traditional income tax consequences of a split-dollar plan are that the employee is considered to be in receipt each year of an amount of taxable economic benefit. The employer cannot deduct any portion of the premium contribution. Death benefits from a split-dollar plan, both the employer's share and the employee's beneficiary's share, are generally income tax free.

What is the unique feature of a Keogh Plan when compared with qualified plans adopted by corporations?

The unique feature of a Keogh plan, when compared with qualified plans adopted by corporations, is that the Keogh plan's contribution on behalf of the owner is based on earned income as opposed to compensation. Earned income is defined as the self-employed individual's net income from business after all deductions, including the deduction for the Keogh Plan contributions.

Using the appropriate life insurance products in a qualified plan can provide unpredictable costs for the employer. State True or False.

The use of appropriate life insurance products for funding a qualified plan can provide extremely predictable plan costs for the employer.

The waiting period that must be satisfied for Social Security disability benefits is

The waiting period must be 5 full months of disability with benefits payable in the sixth full month of a person's disability.

Which of the following describes a method of ownership of a life insurance policy subject to a split-dollar arrangement?

There are two methods of arranging policy ownership under a split-dollar plan. They are the endorsement method and the collateral assignment method.

Which of the following statements is NOT correct regarding a defined benefit pension plan?

These plans are funded actuarially, which means that, for a given benefit level, the annual funding amount is greater for employees who are older at entry into the plan, as the time to fund the benefit is less in the case of an older entrant. The required employer contribution may vary year to year.

Penalty or haircut provision Suspension of participation provision Hardship withdrawal provision

This provision has multiple withdrawal reasons stated, including the request of the participant, but there are penalties for early withdrawal. This provision temporarily stops the employee's participation in the plan for a period such as six months after withdrawing money from the plan. This provision provides withdrawal in case of death, disability or financial emergencies. Rules of Section 401(k) do not apply for this provision.

What are the rights of a minority shareholder's beneficiaries in a closely held corporation? (Select all that apply)

Though the minority shareholder's beneficiaries cannot exercise control in the management of the corporation, they may be able to render life miserable for the survivors. They have rights such as being entitled to a proportionate share of dividends, to examine the corporate records with legitimate reason, and generally to participate in all shareholder activities. The majority shareholder's beneficiaries can enforce their wills on the surviving shareholders, whereas minority shareholder's beneficiaries generally cannot.

Which of the following are ways savings are encouraged through the tax system? Click all that apply.

Three ways saving is encouraged through the tax system are: Deferral of tax on capital gains until realized Potential exclusion of a base amount of the capital gain of the sale of residence Deferral of tax and other benefits for qualified retirement plans and other tax advantaged plans

This year, Tim earned $10,000 in January but did not have earned income subject to Social Security taxes the balance of the year. How many Social Security credits does Tim earn this year?

Tim earns the maximum of 4 credits this year. One credit is earned in 2022 for each $1,510 of compensation subject to Social Security taxes up to a maximum of 4 for the year.

Jane is looking for annuity that may outpace inflation and will guarantee payments for at least 10 years. What type of annuity and payout option should she choose?

To outpace inflation, a variable annuity has a better opportunity to outpace inflation than a fixed annuity because of the investment in the separate account. To ensure that Jane receives payments for 10 years, she must select a certain period payout option. Jane could select a single life payout option. However, depending upon her death she may receive more or less than 10 years.

Under a Section 125 plan (a cafeteria plan), "qualified benefits" - include all of the following except? (Select all that apply)

Under Code Section 125 and its regulations, only certain qualified benefits can be made available. Cash and most tax-free benefits such as medical expenses and disability income insurance are part of the basic benefit package, but the plan excludes scholarships and fellowships under Section 117 and retirement benefits such as qualified or nonqualified deferred compensations.

Which of the following would be taxable to an employee recipient under Code Section 132? (Select all that apply)

Under Code Section 132, benefits that are often referred to as de minimis fringe benefit, states that property or services provided to the employees are not taxable if they are too small to be accountable. IRS regulations have provided guidelines with respect to this so that employees cannot misuse the term "small".

Under ERISA, what are the two types of employee benefit plans? Click all that apply.

Under ERISA, employee benefit plans are divided into two types: pension plans and welfare plans.

Each of the following statements regarding a group carve-out life insurance plan is correct EXCEPT:

Under a carve-out plan, coverage may vary for each participant.

Under an entity purchase buy-sell agreement funded with life insurance, when is the buy-sell purchase price established?

Under an entity purchase buy-sell agreement the buy-sell purchase price is established by agreement between the company and the insured/owner at the time the buy-sell agreement is agreed upon.

What is the longest waiting period permitted for full vesting for matching employer contributions?

Under current law, the longest wait permitted for full vesting of matching employer contributions is six years under the graduated two-to-six-year vesting schedule.

Under which method for split-dollar life insurance does the employee own the policy and is responsible for premium payments?

Under the collateral assignment method for split-dollar life insurance, the employee owns the policy and is responsible for premium payments.

Under the endorsement method for split-dollar life insurance, who owns the policy and is responsible to the insurance company for premium payments?

Under the endorsement method for split-dollar life insurance, the employer owns the policy and is responsible to the insurance company for paying the entire premium.

Under the excess method for integration with Social Security, what is the maximum excess contribution rate to the plan if the base contribution percentage is 6%?

Under the excess method, the maximum excess contribution rate for compensation above the compensation threshold is the lesser of: 2x the base contribution rate or the base contribution rate plus 5.7%. In this example, the base rate + 5.7% (6% + 5.7% = 11.7%) is the maximum excess contribution.

In a sole proprietorship, who is obligated to liquidate the business upon the death of the proprietor?

Upon the death of the proprietor, the proprietor's personal representative generally is obligated to liquidate the business. The personal representative is usually named in the last will and testament of the proprietor and could be the spouse or heirs of the proprietor. In a sole proprietorship, a partner does not exist.

Civil Rights Laws State Legislation General Counsel Memorandum Section Field Service Advice

Used to prohibit employment discrimination on the basis of race, religion, sex, or national origin. Provides room for issues that cannot be dealt with by ERISA. It is an internal IRS document prepared for its own staff's guidance in administering the Code. It is an IRS document prepared for internal use within the IRS.

In a split-dollar life insurance plan, two parties typically divide or "split'' the responsibilities and the rights to which of the following?

Usually split-dollar plans involve a splitting of premiums, death benefits, and/or cash values, but they may also involve the splitting of dividends or ownership.

A qualified plan receives tax benefits that are not available for a nonqualified deferred compensation plan. Which of the following is NOT one of the tax features of a qualified plan?

Vested employees do not pay tax on the employer contributions until they are withdrawn from the plan.

Which of the following employees is NOT considered a highly compensation employee under qualified plans rules if the employer makes the "top 20%" election?

When an employer has made the top 20% election, only employees with income more than $130,000 (2021) who are ranked in the top 20% based on compensation are considered HCE. Any employee with more than 5% ownership is automatically considered HCE, regardless of compensation.

Ted, age 45, is employed by a municipality that sponsors a government Section 457 plan with a normal retirement age of 65. His monthly salary is $7,000. Ted also works part-time for a for-profit company and earns $25,000 annually. The company sponsors a Section 401(k) plan. What is the maximum amount Ted can defer into the Section 457 plan for 2022?

While contributions to a government Section 457 plan are not aggregated with contributions to another plan in which the taxpayer participates, this question is testing only the maximum contribution to the Section 457 plan. For Ted, the maximum contribution is $20,500 (2022)

Which of the following business expenses is NOT covered under a business overhead disability insurance policy?

While employee salaries are covered, owner salary is not covered under a business overhead disability insurance policy.

Assuming the 5-year holding period has been met, when are withdrawals from a Roth IRA tax-free in their entirety? Click all that apply.

Withdrawals are tax-free in their entirely in a Roth IRA: After a five-year wait, and either: Upon death or disability First time home-buying expense After the age of 59 ½

Flexible spending accounts (FSAs) minimize employee outlay by converting after-tax employee expenditures to before-tax expenditures.

Without a flexible spending account (FSA), an employee pays for allowable expenses, such as medical plan deductibles with after-tax dollars. A FSA allows these same expenses to be paid for with before-tax dollars.

Each of the following is a question that should be answered when planning for retirement in 15 years EXCEPT:

"What is my current budget" is not a question considered in retirement planning for the future. Cash flow will be important during retirement but is not typically a question addressed among the other key questions when retirement is still 15 years in the future.

Using the table below, identify the Highly Compensated Employees (HCEs) at BIF-Brews, Co. Employee % Ownership Annual Compensation (2021) Adam 3% $225,000 Brendan 6% $75,000 Jerry 3% $115,000 Mike 5% $135,000

A Highly Compensated Employee (HCE) is: More than a 5% owner or Compensated in excess of $130,000 (2021) (indexed) As a result, Brendan (6%) and Mike (5%) meet the 5% ownership standard. Adam is considered a HCE due to his compensation in excess of $130,000 in 2021 (i.e., $225,000).

What are the main advantages of the Roth IRA? (Select all that apply)

A Roth-IRA account allows contributions of after tax money to be deposited. These contributions can be withdrawn without taxation or penalty at any time. The earnings on the deposits grow tax sheltered. Untaxed earnings may be withdrawn tax free in the future if certain conditions are met. The Roth-IRA does not provide a tax deduction now but may provide a tax break later, perhaps during retirement. With enough years of growth, a substantial amount of untaxed earnings may be withdrawn tax free, providing a significant income tax advantage during retirement.

Matilda Sinclair is an employee of Maxwell Inc. Her earnings for this year are $48,000 and she participates in a money purchase plan that is integrated with Social Security. The plan provides for employer contributions of 15% of compensation above an integration level of $22,000 and 11% below the level. What is the total employer contribution to Matilda's account this year?

A plan benefit formula that is integrated with Social Security avoids duplicating Social Security benefits already provided to the employee and reduces employer costs for the plan. An integrated formula defines a level of compensation known as the integration level. Employer contributions for compensation above the integration level are of a higher rate than the rate for compensation below the level. Maxwell Inc.'s contribution to Matilda's account would be computed as follows: 11% of the first $22,000 of total compensation: $2,420 15% of $26,000 ($48,000 compensation - $22,000 integration level): $3,900 The employer contribution to Matilda's account this year: $6,320.

Netlink, Inc. is considering the establishment of a qualified plan. Which of the following characteristics suggest the use of a profit sharing plan? (Select all that apply)

A profit sharing plan is appropriate in a company whose profits vary from year to year. It is also offered by employers who want to adopt a qualified plan with an incentive feature as well as to supplement an existing defined benefit plan. It is suitable if many employees are relatively young and have substantial time to accumulate retirement savings. Therefore, a profit sharing plan would not be fitting for older employees who want to be assured of contributions or benefits at retirement.

A loan from a qualified plan to a participant must meet all of the following requirements to be exempt from treatment as a prohibited transaction except?

A reasonable rate of interest, availability to all participants on a reasonably equivalent basis and adequate securing of loans are three of the requirements for loans to be exempted from the prohibited transaction rules. However, loans may exceed $10,000, but cannot exceed $50,000.

The contribution levels allowed under a target or age-weighted defined contribution plan:

A target or age-weighted defined contribution plan formula for annual employer contributions or allocations to participant accounts is based on both the participant's compensation and age on entering the plan. Relating allocation percentages to age results in higher allocation for older plan entrants. Defined benefit plans have a recurring annual contribution obligation. Therefore, their contribution levels are higher than a target or age-weighted plan.

Which of the following is NOT an acceptable vesting schedule for a top-heavy defined benefit plan?

A top-heavy defined benefit plan must use accelerated vesting. A 3-to-7-year graded vesting schedule, while acceptable for a defined benefit plan that is not top-heavy, is not acceptable if the defined benefit plan is top-heavy.

Salary continuation formula Salary reduction formula Excess benefit plan Stock appreciation rights

A type of nonelective nonqualified deferred compensation plan that provides a specified deferred amount payable in the future. A plan that involves an elective deferral of a specified amount of the compensation that the employee would have otherwise received. A plan that provides benefits only for executives whose annual projected qualified plan benefits are limited under the dollar limits of Code Section 415. A plan that gives an employee a stake in a company's growth (as reflected in its stock price) without actually investing in the company's stock.

The savings plan after-tax employee contribution approach is used more often as an add-on to a Section 401(k) plan. State True or False.

Although savings plans with only after-tax employee contributions and employer matching contributions were very popular in the past, the after-tax employee contribution approach is used more often as an add-on to a Section 401(k) plan.

Investment strategy must be formulated keeping in mind what factors? (Select all that apply)

Among other things, an investment strategy must take into consideration the time horizon, risk tolerance, and tax considerations. It is dangerous to have too much in stocks and bonds and not enough in emergency reserves as liquid assets. The strategy for investment of retirement funds must reflect the investment time horizon. As the time for retirement approaches, funds must be gradually shifted to less risky investments. An increase in inflation rates will affect the value of stocks and bonds. Selling stocks and mutual funds will result in having to pay capital gains taxes. The chief goal of the investment strategy must be to allow a person to earn enough on retirement savings to offset inflation and not to maximize earnings with risky investments..

Which of the following is considered constructive receipt for income tax purposes?

An amount is treated as received for income tax purposes, even if it is not actually received, if it is credited to the employee's account, set aside, or otherwise made available. Constructive receipt does not occur if the employee's control over the receipt is subject to a substantial limitation or restriction.

Jennifer is the owner of a software corporation. As an employer she would like to enhance the retirement benefits received by Rob, a senior executive in the company. She consults her financial consultant, Randolph, and inquires about nonqualified plans. He tells her that it is indeed a good idea to opt for a nonqualified plan. He also gives her the various other instances where she can use the plan. Which situations would Jennifer be able to use the plan to benefit both Rob and her? (Select all that apply)

As an employer, Jennifer can use the deferred plan to her advantage, as she can provide additional benefits to Rob within the company's budget. She need not use the same benefit program for all other employees, that is, the plan gives her the flexibility to decide whom to include for the benefit plan. Also, by using the deferred plan she can make sure that Rob is happy with the working conditions in the company. However, Jennifer will not receive a tax deduction in the year that Rob receives the compensation. The deduction will be deferred until the year that the income is taxable to Rob, and this could take up to 30 years or more.

Which of the following retirement savings strategies, with amounts invested at the end of each year, produces the largest lump sum at age 65 assuming an 8% annual return?

At age 35, investing $6,000 annually results in a lump sum at age 65 of $679,699. N = 30, I/YR = 8, PV = 0, PMT = -$6,000, Solve for FV = $679,699 At age 35, investing $10,000 annually for 10 years with no further additional investments results in a lump sum at age 65 of $$675,212. Step 1: N = 10, I/YR = 8, PV = 0, PMT = -$10,000, Solve for FV = $144,865.62; Step 2: N = 20, I/YR = 8, PV = -144,865.62, Solve for FV = $675,212 At age 45, investing $10,000 annually results in a lump sum at age 65 of $457,620. N = 20, I/YR = 8, PV = 0, PMT = -$10,000, Solve for FV = $457,620 At age 45, investing $100,000 with no additional investments results in a lump sum at age 65 of $466,096. N = 20, I/YR = 8, PV = -$100,000, Solve for FV = $466,096

Brenda works for ABC Inc. and contributes $1,500 to her qualified plan during her first year of participation. The ABC Inc. plan has a 3-year cliff vesting schedule. How much of her contributions can she take with her if she leaves ABC Inc.?

Brenda can take 100% of her contributions from the plan because employee contributions are always 100% vested. Employer contributions, on the other hand, follow the vesting schedule.

Bruce, as an employer, would like to provide Tom with retirement benefits. He is looking at life insurance as a viable option. In which of the following situations could he use the split dollar insurance plan? (Select all that apply)

Bruce could use the split dollar life insurance option if he wants to provide Tom retirement benefits without having to spend too much on it. This option would be ideal if Tom is in his 30s, 40s or early 50's, as the plan requires a reasonable duration so as to build up adequate policy cash values and because the cost to the executive, that is the P.S. 58 or Table 2001 cost, can be excessive at later ages. He can also use it if a preretirement death benefit for Tom is a major objective. He will use this option if he is looking for a totally selective executive fringe benefit. The other situation where Bruce can use this option is when he wants to make it easier for shareholder-employees like Tom to finance a buyout of stock under a cross purchase buy-sell agreement, or to make it possible for non-stockholding employees to effect a one-way stock purchase at an existing shareholder's death.

To whom does the Social Security Administration currently mail annual statements?

Currently, the Social Security Administration mails annual statements to Individuals aged 60 and older who do not have a "my Social Security" account online who are paying into the Social Security system.

A A defined benefit plan is a qualified plan that provides a specified benefit level at retirement. Which of the following is not a characteristic of defined benefit plans? (Select all that apply)

Defined benefit plans are complex to design and difficult to explain to employees. Moreover, only the employer assumes the risk of bad investment results in the plan fund. The employer must contribute to the plan fund annually. The contribution amount is based on actuarial assumptions. The employer and the PBGC guarantee certain benefit levels within specified limits.

Employees who participate in savings plans do so on a voluntary basis, contributing a percentage of compensation. Which of the following accurately describes savings plans?

Employer contributions to savings plans are deductible so long as the plan remains qualified, but employee contributions to the plan are not tax deductible. The savings plan must meet an actual contribution percentage test to be deemed nondiscriminatory.

Which of the following methods to ultimately fund promised benefits to an employee under a nonqualified plan may cause the plan to be considered "funded?"

Employer involvement in securing a third-party guarantee raises the possibility that the guarantee will may the plan to be deemed formally funded for tax purposes.

Governmental employers may offer 401(k) plans as a benefit to their employees? State True or False.

False

Gerardo has worked for FIX Company for 3 years and has decided to leave. The company elected a 5-year cliff vesting schedule. His Defined Benefit Plan will be portable and will roll over to his next employer.

False. Employees who leave before retirement may receive relatively little benefit from the Defined Benefit Plan. The plan generally lacks portability in that when an employee changes jobs, it usually cannot go with him or her. In addition, since FIX Company has a 5-year cliff vesting schedule, Gerardo would not be vested at all.

Which of the following provisions would violate the Code's vesting requirements for qualified retirement plans?

In a Defined Contribution plan, the vesting schedule must not exceed the Top Heavy vesting schedules: 2-6 year graded or 3 year cliff. A Defined Benefit plan may use 3-7 year graded and 5 year cliff. Therefore, in a Defined Benefit plan, the employee must be at least 60% vested after five years of service which increases by 20% per year until full vesting in seven years.

Under which type of sponsored plan is the employee's pension benefit determined by a formula that takes into account years of service to the employer and, in most cases, wages or salary?

In a defined-benefit plan, the employee's pension benefit is determined by a formula that takes into account years of service to the employer and, in most cases, wages or salary. The amount the employee receives is defined for him or her. Most common formulas used to determine the benefit are flat amount, flat percentage and unit credit.

Samantha Kruger asked you, "What is the best way to save for retirement?" How would you answer her?

In retirement planning, the fact that taxes affect personal financial decisions cannot be overstressed. Of the many options available, using a tax-deferred retirement plan is most beneficial because they allow investment earnings to go untaxed until the earnings are removed at retirement. Social Security, an employer's retirement plan and an IRA may or may not be taxed.

Which of the following requirements must be met to qualify for a mortgage loan exception? (Select all that apply)

It is necessary for the loan to be secured by a mortgage on the employee's new residence. The loan has to be compensation-related. Also, it must be a demand or term loan. The new residence has to be acquired in connection with the transfer of the employee to a new principal place of work. It must meet the distance and time requirements for a moving expense deduction under Code Section 217. There is no obligation for the loan to be payable in full in 15 days after the old principal residence is sold. The rule that the old residence must not be converted to business or investment use does not apply here.

Which of the following are considered advantages of a nonqualified plan? Click all that apply.

Nonqualified plans have many advantages, such as a flexible design, minimal government regulatory requirements, deferral of employee taxes, use by the employer as "golden handcuffs" that help bind the employee to the company, and some assets set aside in some informal arrangements are available to use for corporate purposes.

Which of the following are features of nonqualified plans? (Select all that apply)

Nonqualified plans provide employees additional retirement benefits. The plans are not qualified under ERISA requirements. As a result, the employer has freedom while planning the program. The employer can decide who is to be included in the plan. He may want to include only highly compensated employees or members of management. Plan benefits for these groups are forfeitable in full at all times.

Typically, a split dollar plan involves an executive purchasing life insurance and naming the employer as beneficiary and owner of the contract.

Normally, the life insurance policy is owned by the employer who also controls and owns the cash value as well as enough of the death benefit to cover expenses if the executive dies. The balance of the death benefit goes to the executive's beneficiary.

Russell and Charmin have current living expenses that equal $57,000 a year. Assuming 80% replacement, estimate the amount of income they will need to maintain their level of living in retirement. Assume their average tax rate will be 13%, not including inflation.

Russell and Charmin will need 80% of their current living expenditures of $57,000. Tax must then be added at the estimated average tax rate of 13%, using the formula retirement income/(1-tax rate). $57,000 x 0.80 = $45,600 $45,600/(1-0.13) = $45,600/0.87 = $52,413.79

Lou has taken advantage of his employer's Roth 401(k) plan. He will contribute $11,000 per year. If, in ten years at age 62, his account balance is $165,000 and Lou decides to withdraw his entire account balance, what will be the taxable portion of that withdrawal?

Since Lou satisfied the five year and is withdrawing his gains following age 59 ½, all gains are tax-free. Contributions are always tax free since they were made with after tax dollars.

Match the corresponding types of annuities on the left with the descriptions on the right.

Single Life: Provides a set monthly payment for the person's entire life. Annuity for Life with Period Certain: A person will receive payment for life, but if he or she dies before the end of certain period, payments will continue to beneficiaries until the end of the period. Joint and Survivor: Provides payments over the life both the annuitant and his/her spouse.

Which unexpected circumstances resulting in financial hardship are considered an unforeseeable emergency? (Select all that apply)

Sudden and unexpected illness or accident or property loss due to casualty are considered unforeseeable emergencies.

Audrey is an employee of the Spencer Corporation. She borrows $250,000 from her employer. The applicable federal rate of interest for the first year is $16,000. The actual interest under the loan agreement is only $7,000. This loan results in additional taxable compensation income to Audrey for the first year of $9,000. What is the amount deductible by her employer?

The amount deductible by Audrey's employer is $9,000. This would be the difference between the applicable federal rate of interest less the actual interest under the loan agreement, that is, $16,000 less $ 7,000. The employer is treated as if it paid the additional compensation to the employee.

Which of the following factors determines the appropriate face amount of insurance coverage? (Select all that apply)

The appropriate face amount of insurance coverage is determined by the following factors: employer's after-tax costs of each participant's retirement benefits, premium funding requirements, the present value of any survivor benefits and, if preferred, the cost or time value of money.

An employer wishes to establish a qualified plan in which the annual contribution is a percentage of each participant's compensation. The work force is large, middle-income and mostly young to middle -aged. Assuming one of the employer's goals is to provide a plan backed by the PBGC, which of the types of plans would be the first recommendation?

The cash balance pension plan is most appropriate if the work force is large and the bulk of the employees are middle income, when compared with the defined benefit pension plan, 401(k) plan or the profit sharing plan.

A client's current annual salary is $100,000. If the client wishes to replace 80% of their current salary, in today's dollars when they retire in 20 years, what capital amount will be needed to accumulate by the first day of retirement if the expected rate of inflation will be 2%, the expected rate of investment returns is 5%, and the client expects to live 25 years in retirement?

The client will need to accumulate $2,144,922 by the first day of retirement to support the retirement income goal. Step 1: 100,000 x 80% = 80,000 80,000 [+/-] PV 20, N 2, I/YR Solve FV 118,875 Step 2: BEGin mode 118,876 PMT 25, N [(1.05 ÷ 1.02) - 1 x] = 2.9412 I/YR 0, FV Solve PV 2,144,922

Christopher is an executive working in a software company. One day he approaches Veronica, the company's financial advisor, because he needs some information on the loan programs that the company runs for its employees. During the conversation Veronica tells him the situations in which the company may sanction a loan, such as a loan for a new home if the company moves him to a different city. For which of the following situations would Christopher be sanctioned a loan? (Select all that apply)

The company will sanction a loan to pay for Michael's education and to clear the medical expenses that the family had to incur during Christopher's mother's yearlong battle against cancer. It would also sanction a loan if he wanted to buy a life insurance policy. The company will, however, reject the loan for the house in Nevada and for settling credit card dues. According to the company policy, only if Christopher were to move from his current office to a different city would a loan be sanctioned for a new home.

What is the current maximum covered compensation limit in calculating benefits in a qualified plan?

The current (2022) maximum covered compensation limit in calculating benefits in a qualified plan is $305,000.

Which of the following questions are relevant to setting retirement goals? (Select all that apply)

The income tax bracket of children of the retiree is irrelevant to setting retirement goals. The other questions must be answered to assess the financial goals that must be set before planning for retirement..

A qualified retirement plan must meet stringent Code requirements. Which of the following would be most likely to result in plan disqualification?

The plan would be disqualified if a five-year waiting period is imposed for plan participation. It cannot require more than one year of service for eligibility. The only exception is that it can require a waiting period of two years, if the plan provides 100% vesting upon entry.

Jack, age 25, has been working for Boat Company for two years and is now eligible to participate in Boat Company' qualified plan. Boat Company must 100% vest Jack in his plan.

The plan's waiting period can be up to two years if the plan provides immediate (100%) vesting upon entry.

The maximum annual deductible employer contribution to a defined benefit plan is

The required contribution to the plan must bring the plan assets up to the sum of the funding target and the target normal cost. In other words, the plan assets must equal the accrued benefits that the plan must pay.

Under which of the following hypothetical situations would a cash balance pension plan be indicated?

The retirement benefit may be inadequate for older plan entrants. A cash balance pension plan is appropriate when employees are relatively young and the work force is large and the bulk of the employees are middle-income. It is indicated when the employer has an existing defined benefit plan and wishes to convert to a plan that provides a more attractive benefit for younger employees and lower costs for older employees.

Senior executive Emily Thompson is covered under her company's incentive stock option plan. She was granted an incentive stock option in 2020 to purchase 100 shares company stock for $150 per share. In 2021, she purchases 100 shares for a total of $15,000. The fair market value of the 100 shares in 2021 is $25,000. What is the regular income tax treatment of Emily's purchase for 2021?

There is no gain recognized for regular income tax purposes upon exercise of an incentive stock option. The tax treatment of any gain will be determined when the shares acquired are subsequently sold.

Which of the following vesting schedules is NOT permitted for a defined contribution pension plan?

Three-to-five-year graded vesting is not permitted for a defined benefit purchase pension plan. Three-to-five-year cliff vesting is permitted only in a defined benefit plan.

Each of the following is correct regarding nonqualified plans EXCEPT:

Typically, nonqualified plans do not provide an immediate tax deduction for the sponsoring company. The company receives a deduction when the employees receive benefits, or otherwise when the funds are made available.

Since eligible 457 plans are a form of nonqualified deferred compensation, these is no statutory limit on the amount of income that participants may defer. State True or False.

Under Section 457 of the IRC, plans that include limits on the amounts deferred are subject to favorable tax treatment. These plans are generally referred to as eligible Section 457 plans.

When planning an annuity payout, there are several options from which to choose. Which option provides the retiree's beneficiary with benefits until the end of a specified period, if the retiree dies within that period?

Under an annuity for life with certain period a person will receive payments for life, but if he or she dies before the end of a certain period, payments will continue to beneficiaries until the end of that period. The single life annuity provides a set monthly payment for the person's entire life. The joint and survivor annuity provides payments over the life of both the annuitant and his/her spouse. The lump sum payments are not annual payments.

Under the excess method for integration with Social Security, what is the maximum excess contribution rate to the plan if the base contribution percentage is 5%?

Under the excess method, the maximum excess contribution rate for compensation above the compensation threshold is the lesser of: 2x the base contribution rate or the base contribution rate plus 5.7%. In this example, the lesser of 2x the base contribution rate (2 x 5% = 10%) is the maximum excess contribution.

Chairman and owner of Winger Corporation, Jack Winger, would like to reward Denise for her hard work and perseverance. In 2020, she was granted non-qualified stock options of 1,000 shares of the company stock at $20 per share, the market value for the shares that year. In 2021, Denise purchases 500 shares for a total of $10,000. If the fair market value of the shares at exercise is $13,000, what is the regular income tax treatment of Denise's purchase for 2021?

Upon exercise of nonqualified stock options the "bargain element," the difference between the exercise price and the fair market value, is recognized as ordinary income.

Which of the following are not advantages of a Section 401(k) plan? (Select all that apply)

With a Section 401(k) plan, account balances at retirement age may not provide adequate retirement savings for employees who entered the plan at later ages. Further, due to the ADP nondiscrimination test, a Section 401(k) plan can be relatively costly and complex to administer. The advantages are that it allows employees to choose the amount they wish to save. It also permits loans and hardship withdrawals and can be funded entirely through salary reductions.


Conjuntos de estudio relacionados

Chapter 13 - Supply Chain Management & Marketing Channels

View Set

Ch. 1: Perspectives on Maternal, Newborn, and Women's Health Care

View Set